Infectious diseases II

¡Supera tus tareas y exámenes ahora con Quizwiz!

Which of the following describes a patient with acute otitis media who meets criteria for initial management with 48-72 hours of observation? (Select ALL that apply)

B. 12 months old, unilateral infection, maximum temperature 101° F Acute otitis media (AOM) is the most common indication for antibiotic treatment in children. Although antibiotics can accelerate pain relief and prevent complications (eg, tympanic membrane perforation), their use is associated with adverse effects (eg, vomiting, diarrhea) and can promote bacterial resistance. In addition, some cases of AOM resolve without antibiotic treatment. For these reasons, observation and symptomatic care (eg, analgesia) for an initial 48-72 hours is an option in patients with nonsevere symptoms, defined as a temperature below 102.2°F (39°C), ear pain for less than 48 hours, and either of the following criteria: Age 6-23 months with unilateral infection (Choice B) Age ≥ 2 years with unilateral or bilateral infection (Choice D) (Choice A) Because infants age ≤ 6 months have a developing immune system and higher risk for complications, and because they were excluded from studies assessing the role of observation, they should be treated immediately with antibiotics, regardless of the severity or location (eg, bilateral, unilateral) of the AOM infection. (Choices C and E) Children younger than age 2 years with bilateral AOM should be treated immediately with antibiotics. In addition, a temperature above 102.2°F (39°C) in children of any age indicates severe symptoms and can serve as an independent criterion for antibiotic treatment without an initial observation period. Things to remember:A 48-72 hour observation period without antibiotics can be considered for the initial management of acute otitis media in children with nonsevere symptoms [temperature less than 102.2°F (39°C) and ear pain less than 48 hours] and age 6-23 months (unilateral infection only) or age 2 years or older (unilateral or bilateral infection).

A 78-year-old female is admitted to the hospital after a fall. She is scheduled to undergo total hip arthroplasty for a fractured femur and will receive cefazolin for surgical site infection prophylaxis. What is the recommended timing of the first dose of cefazolin for this procedure?

A. 1 hour before the first incision Antibiotics, when timed appropriately, reduce the incidence of surgical site infections. Successful prophylaxis requires: Delivery of antibiotic concentrations to the operative site before contamination can occur (eg, prior to the first incision), such that peak tissue concentrations exceed the minimum inhibitory concentration for the probable organisms associated with the procedure (eg, staphylococci). Antibiotic redosing during the procedure based on the half-life of the chosen drug and the expected duration of the procedure (eg, longer than 4 hours). A total duration less than 24 hours after the procedure for most surgeries. A duration longer than 24 hours does not result in further reductions in surgical site infections but does increase the risk of adverse events from prolonged antibiotic exposure (eg, Clostridioides difficile infections). Beta-lactam antibiotics (eg, cefazolin) should be infused within one hour before the first incision due to their shorter administration times and half-lives. Conversely, vancomycin and fluoroquinolones should be initiated 2 hours before the first incision because of their longer infusion times and half-lives (Choice C). (Choices B and D) To be effective, the first dose of antibiotics must be given prior to (not after) the first incision so that adequate tissue concentrations are reached and can provide protection against contamination. (Choice E) Cefazolin has a half-life of approximately 2 hours; infusing it 4 hours before the first incision would result in inadequate tissue concentrations (ie, 75% of the dose would be eliminated). Things to remember:Appropriate timing of prophylactic antibiotics is necessary to reduce the incidence of surgical site infections. Beta-lactams should be infused within 1 hour before the first incision; vancomycin and fluoroquinolones should be initiated 2 hours before the first incision.

A 22-month-old male is brought to the office due to a fever of 102.6°F (39.2°C), irritability, and tugging of his right ear for the past 2 days. The patient is diagnosed with a first occurrence of acute otitis media. He has no known drug allergies. Which treatment is most appropriate for this patient at this time?

A. Amoxicillin Acute otitis media (AOM), typically caused by Streptococcus pneumoniae, Haemophilus influenzae, or Moraxella catarrhalis, is the most common infection requiring antibiotic treatment in children. Some cases of AOM can initially be managed with 48-72 hours of observation; however, this patient age < 2 years with fever > 102.2°F (39°C) and ear-tugging (a symptom of pain) for > 48 hours should be treated with antibiotics. The first-line treatment for AOM is high-dose (90 mg/kg/day administered in 2 divided doses) amoxicillin or amoxicillin/clavulanate. Amoxicillin/clavulanate is preferred when there is risk for beta-lactam resistance (eg, amoxicillin use in the past 30 days). When used, the formulation containing the least amount of clavulanate should be selected to reduce gastrointestinal distress (eg, diarrhea). (Choices B and E) Cephalexin (a first-generation cephalosporin) and penicillin V potassium are not indicated for the treatment of AOM due to limited activity against H. influenzae and resistant strains of S. pneumoniae. Second- and third-generation cephalosporins (eg, cefuroxime, cefdinir) are alternatives to amoxicillin if the patient has a mild penicillin allergy. (Choice C) Doxycycline (a tetracycline) should not be used in children age < 8 years due to the risk of diminished bone growth and irreversible staining of the teeth. (Choice D) An initial 48- to 72-hour observation period without antibiotics is appropriate only for nonsevere AOM [eg, ear pain < 48 hr, temperature < 102.2°F (39°C)] in children age 6-23 months with unilateral infection or those age > 2 years with unilateral or bilateral infection. Things to remember:High-dose (90 mg/kg/day administered in 2 divided doses) amoxicillin or amoxicillin/clavulanate is the first-line treatment for acute otitis media.

JM is a 36-year-old female who visits her primary care provider reporting a 3-day history of fatigue, chills, and productive cough. On physical examination, she is noted to have rales in the right lower lobe. Past Medical History: allergic rhinitis, congenital long QT syndrome, migraine Allergies: no known drug allergies Medications:Flonase 1 spray each nostril once dailyIbuprofen 200 mg PRN headachesZyrtec 10 mg QHS Vital Signs: BP 130/90 mmHg, HR 88 bpm, RR 16 bpm, O2 sat 95% on room air, T 102°F (38.9°C) Diagnostic Tests: chest x-ray pending Which antibiotic is most appropriate to empirically treat community-acquired pneumonia in this patient?

A. Amoxicillin The outpatient treatment of community-acquired pneumonia (CAP) in patients without significant comorbidities (eg, chronic heart, lung, liver, renal disease) consists of monotherapy with either high-dose amoxicillin (1 gram TID), doxycycline, or a macrolide (eg, azithromycin, clarithromycin). All these agents provide adequate activity against the most common CAP pathogens, including Streptococcus pneumoniae and Haemophilus influenzae. Macrolides and doxycycline have added activity against atypical organisms (eg, Mycoplasma pneumoniae, Chlamydia pneumoniae, Legionella spp.) whereas amoxicillin lacks atypical coverage. (Choices B and E) To minimize the risk of triggering a ventricular arrhythmia, medications associated with QT prolongation (eg, macrolides, fluoroquinolones) should be avoided with any of the following: Evidence of QT prolongation on electrocardiogram (ECG) Uncorrected electrolyte abnormalities (eg, hypokalemia, hypomagnesemia) Concurrent use of more than one drug that can prolong the QT interval Congenital long QT syndrome (Choice C) Cephalexin (an oral first-generation cephalosporin) has activity against streptococci, methicillin-susceptible Staphylococcus aureus (MSSA) and some activity against the gram-negative rods Proteus, Escherichia coli, and Klebsiella (commonly abbreviated as PEK). Cephalexin is commonly used for uncomplicated urinary tract infections and skin and soft tissue infections (eg, nonpurulent cellulitis without concern for methicillin resistance). (Choice D) Linezolid (an oxazolidinone) has activity against gram-positive bacteria only and is commonly used to treat infections caused by methicillin-resistant S. aureus (MRSA) and vancomycin-resistant enterococci (VRE). It does not have activity against atypical organisms. Things to remember:Monotherapy with high-dose amoxicillin, doxycycline, or a macrolide is recommended for the outpatient treatment of community-acquired pneumonia in patients without comorbidities. Macrolides and fluoroquinolones should be avoided in patients with QT prolongation risk when safer treatment options are available.

DP is a 49-year-old male who visits his dentist for severe tooth pain. After examination, the dentist informs him that he needs a tooth extraction. Past Medical History: prosthetic mitral valve replacement, hypertension, myocardial infarction, dyslipidemia Allergies: no known drug allergies Home Medications:Amlodipine 10 mg PO dailyWarfarin 6 mg PO daily or as directedHydrochlorothiazide 25 mg PO dailyAtorvastatin 80 mg PO dailyNitroglycerin 0.4 mg SL PRN chest painMetoprolol succinate 100 mg PO dailyLisinopril 5 mg PO dailyAspirin 81 mg PO daily Vital Signs: BP 120/80 mmHg, HR 62 bpm, RR 16 bpm, T 98.5°F (36.9°C), Ht 5′ 9″, Wt 180 lbs Which antibiotic treatment is preferred in this patient prior to the procedure?

A. Amoxicillin 2 grams PO once Dental procedures involving the manipulation of gingival tissue or the periapical region (near the root of the tooth) can lead to translocation of oral bacteria (eg, viridans streptococci) into the bloodstream and subsequent infective endocarditis (IE). Antibiotic prophylaxis is therefore indicated for high-risk patients, including those with a prosthetic heart valve, history of IE, or history of heart transplant with abnormal valve function. Procedures requiring prophylaxis include tooth extractions, root canals, and tooth abscess drainage. The first-line treatment for IE prophylaxis is amoxicillin 2 grams PO. Options for patients with a penicillin allergy include a macrolide (azithromycin or clarithromycin) or doxycycline. Antibiotics should be administered as a single dose 30-60 minutes before the procedure to ensure systemic concentrations are adequate to prevent any transient bacteremia. (Choice B) Intramuscular or intravenous ampicillin is an option for patients unable to tolerate oral medications; the correct dose is 2 grams. (Choice C) Azithromycin 500 mg (not 1,200 mg) is recommended in patients with a penicillin allergy. Macrolides are not otherwise preferred over amoxicillin because of rising streptococcal resistance. (Choice D) Clindamycin is not recommended for IE prophylaxis due to its propensity for adverse effects, including Clostridioides difficile. (Choice E) Antibiotics are indicated because the patient has a high-risk condition (prosthetic mitral valve) for developing IE after a dental procedure. Things to remember:Antibiotic prophylaxis before a dental procedure is indicated for patients at high risk for developing endocarditis (eg, prosthetic heart valve, history of endocarditis). Oral amoxicillin 2 grams is the preferred treatment for patients without a beta-lactam allergy who can take oral medications.

TM is a 32-year-old male who comes to his primary care physician with symptoms of thick, yellow-green nasal discharge, nasal "stuffiness," and a feeling of pressure in his face and ears. He also describes more frequent headaches. The patient initially thought he had a cold and self-treated with OTC medications, but his condition worsened and has persisted for approximately 12 days. TM has no known drug allergies. A decision is made to prescribe antibiotics. Which of the following is a first-line treatment for TM's condition?

A. Amoxicillin/ clavulanate Acute sinusitis (or rhinosinusitis) is diagnosed when the cardinal symptoms of nasal obstruction, purulent nasal discharge, and facial pressure/pain occur for up to 4 weeks. To avoid unnecessary antibiotic use, viral sinusitis should be distinguished from acute bacterial sinusitis (ABS), which is diagnosed with either of the following criteria: Symptoms that persist ≥ 10 days without improvement Severe symptoms (temperature > 102°F, purulent nasal discharge, facial pain) ≥ 3 days Amoxicillin-clavulanate, the first-line antibiotic treatment, targets the common pathogens (eg, Streptococcus pneumoniae, Haemophilus influenzae, Moraxella catarrhalis), including resistant strains. Alternatively, patients with reliable follow-up can receive symptomatic care for up to 7 days, with antibiotics prescribed only if symptoms do not improve. (Choices B and E) Macrolides (eg, clarithromycin) and sulfamethoxazole/trimethoprim are not recommended for ABS because the prevalence of S. pneumoniae and H. influenzae resistance results in a high rate of treatment failure. (Choice C) Clindamycin has activity against gram-positive organisms (eg, streptococci, staphylococci) but no gram-negative activity. If combined with another antibiotic (eg, a third-generation cephalosporin), it can be used to treat ABS in patients with a mild penicillin allergy. (Choice D) Fluoroquinolones (eg, levofloxacin) are a recommended alternative for ABS in patients with a type I hypersensitivity reaction (eg, anaphylaxis) to penicillin, but because of adverse effects (eg, tendonitis, peripheral neuropathy), they have a boxed warning to use last-line for this indication. Things to remember:Acute bacterial sinusitis is suspected when symptoms (nasal congestion, purulent nasal discharge, facial pressure/pain) persist for ≥ 10 days or are severe (eg, temperature > 102°F) for ≥ 3 days. The first-line treatment is amoxicillin/clavulanate, with or without an initial 7-day period of observation and symptomatic care.

PC is a 69-year-old female who comes to the emergency department with increased shortness of breath, productive cough, and increased production of sputum that is yellow/green in color. She has had no hospitalizations or antibiotic use in the past 6 months. Past Medical History: hypertension, chronic obstructive pulmonary disease (FEV1 60%), hyperthyroidism Social History: previously smoked cigarettes (30 pack-year), quit 5 years ago Allergies: no known drug allergies Medications:Amlodipine 10 mg PO dailyRamipril 10 mg PO dailySpiriva Respimat 2 inhalations daily Vital Signs: BP 138/90 mmHg, HR 92 bpm, RR 24 bpm, O2 sat 89% on room air, T 99.7°F (37.6°C) Diagnostic Tests:Chest x-ray: no evidence of pleural effusion or pneumonia Assessment and Plan: COPD exacerbation. Give oxygen via nasal cannula (goal: O2 saturation > 90%), nebulized albuterol treatment, and prednisone 40 mg × 1 dose. Discharge prescriptions written for inhaled albuterol, oral prednisone, and oral antibiotics. Follow-up with primary care provider within 1 week. Which antibiotic is most appropriate to manage this patient's acute condition?

A. Amoxicillin/ clavulanate This patient is experiencing a chronic obstructive pulmonary disease (COPD) exacerbation, defined as an acute worsening of respiratory symptoms when other causes (eg, pneumonia) are ruled out. Exacerbations are commonly triggered by viruses (eg, rhinovirus) but can have a bacterial (eg, Haemophilus influenzae, Moraxella catarrhalis, Streptococcus pneumoniae) or noninfectious (eg, air pollution) etiology. The management of COPD exacerbations includes oxygen supplementation, inhaled bronchodilators (eg, albuterol), and/or systemic steroids (eg, prednisone). Antibiotics (eg, amoxicillin/clavulanate) decrease the risk of treatment failure in patients who meet the criteria in any one of the following categories: Three cardinal symptoms: increased dyspnea, sputum volume, and sputum purulence Increased sputum purulence plus one other cardinal symptom Mechanical ventilation required (Choices B and C) Cefepime (a fourth-generation cephalosporin) and ciprofloxacin both have activity against Pseudomonas aeruginosa, making them too broad-spectrum for patients without specific risks (eg, chronic Pseudomonas colonization, antibiotic use within the past 3 months). In addition, ciprofloxacin lacks activity against S. pneumoniae (ie, it is not a respiratory fluoroquinolone), and cefepime is available only intravenously (eg, not suitable for discharge). (Choice D) Tobi is an inhaled formulation of tobramycin (an aminoglycoside). It is indicated to treat chronic P. aeruginosa infections in patients with cystic fibrosis. (Choice E) Zyvox (linezolid) has activity against only gram-positive pathogens, including methicillin-resistant Staphylococcus aureus, and has no activity against gram-negative pathogens (eg, H. influenzae). Things to remember:Antibiotics are indicated for chronic obstructive pulmonary disease exacerbations when three cardinal symptoms (increased dyspnea, sputum purulence, and sputum volume) are present, two cardinal symptoms are present (including increased sputum purulence), or the patient requires mechanical ventilation. Amoxicillin/clavulanate is the preferred antibiotic.

XW is a 64-year-old male who comes to his primary care provider with a 4-day history of cough with greenish phlegm, fatigue, shortness of breath, and decreased appetite. He has had no recent hospitalizations or antibiotic use. XW does not use tobacco or alcohol. Past Medical History: hypertension, seizures, coronary artery disease, type 2 diabetes mellitus, peripheral neuropathy, peptic ulcer disease Allergies: shellfish (throat tightness, wheezing) Medications: ramipril, aspirin, hydrochlorothiazide, metoprolol, famotidine, topiramate, rosuvastatin, sublingual nitroglycerin PRN Vital Signs: BP 130/80 mmHg, HR 88 bpm, RR 20 bpm, T 100.2°F (37.9°C) Diagnostic Tests Chest x-ray: opacities in the left lower lobe consistent with pneumonia Which outpatient antibiotic regimen is the best option to treat XW's infection?

A. Amoxicillin/ clavulanate and azithromycin The recommended antibiotic regimens for the outpatient treatment of community-acquired pneumonia (CAP) in patients with comorbidities (eg, chronic heart disease) provide adequate activity against common CAP pathogens, including resistant strains of Streptococcus pneumoniae, beta-lactamase producing Haemophilus influenzae, and atypical organisms (eg, Mycoplasma pneumoniae, Chlamydia pneumoniae, Legionella spp.). Selection of a regimen should be based on an individual patient risk-benefit assessment (eg, allergies, drug-disease or drug-drug interactions). In this patient with preexisting peripheral neuropathy and seizures, a beta-lactam (eg, amoxicillin/clavulanate) plus macrolide (eg, azithromycin) is a safer treatment option. Fluoroquinolones (eg, moxifloxacin) should be avoided (Choice D) due to boxed warnings for potentially irreversible peripheral neuropathy, central nervous system adverse reactions (eg, seizures), and risk of tendon rupture. (Choices B and C) Clarithromycin (a macrolide) and doxycycline (a tetracycline) have activity against common CAP organisms (eg, S. pneumoniae, H. influenzae, atypical pathogens) but should be paired with a beta-lactam antibiotic in patients with comorbidities to empirically cover resistant pathogens. Although clindamycin has activity against gram-positive organisms (including streptococci) and anaerobes, it lacks activity against gram-negative pathogens (eg, H. influenzae, M. catarrhalis) and is not an adequate choice in combination with doxycycline. (Choice E) Sulfamethoxazole/trimethoprim is not recommended for CAP because it has unreliable activity against S. pneumoniae and lacks activity against atypical pathogens. It is more commonly used for community-acquired methicillin-resistant Staphylococcus aureus (CA-MRSA) infections and opportunistic infections (eg, Pneumocystis pneumonia). Things to remember:A beta-lactam plus a macrolide or doxycycline is a safer option than a respiratory fluoroquinolone when treating outpatient community-acquired pneumonia in patients with preexisting peripheral neuropathy, central nervous system disorders, or risks for tendon rupture.

A 14-day-old female infant (delivered at term) is brought to the emergency department due to a high fever. She is lethargic, flushed, and difficult to rouse. The parents report that the infant vomited after a feeding approximately 6 hours ago and has been too drowsy to attempt feeding since. In the emergency department, a physical examination is concerning for meningitis; laboratory tests and a lumbar puncture are pending. Which empiric antibiotic regimen should be initiated?

A. Ampicillin + cefotaxime Neonatal meningitis (occurring within 1 month of birth) can result from vertical transmission of maternal genital tract flora (eg, group B streptococcus, Escherichia coli) or can be community- or hospital-acquired. Common presenting symptoms in a neonate include fever (or hypothermia), lethargy, poor feeding, vomiting, and seizures. A lumbar puncture performed to collect and analyze the cerebrospinal fluid (CSF) aids in the diagnosis. Empiric antibiotics that adequately penetrate the CSF and have activity against the most common causative pathogens (group B streptococcus, E. coli, and Listeria monocytogenes) should be initiated promptly. The preferred options include ampicillin plus either cefotaxime or gentamicin. Ampicillin is the treatment of choice for L. monocytogenes; the addition of cefotaxime or gentamicin optimizes activity against streptococci and ampicillin-resistant E. coli. (Choices B, D, and E) Ceftriaxone is contraindicated in neonates because it can displace bilirubin from albumin-binding sites, resulting in kernicterus. Vancomycin is not required for empiric treatment because coagulase-negative staphylococci and methicillin-resistant Staphylococcus aureus are uncommon causes of neonatal meningitis. Meropenem is appropriate when there is a concern for multidrug-resistant gram-negative pathogens (eg, prolonged hospitalization in a neonatal intensive care unit with a high incidence of resistance) or if the CSF culture indicates the presence of an extended-spectrum beta-lactamase-producing organism. (Choice C) Although cefotaxime combined with gentamicin will adequately cover streptococci and E. coli, this regimen lacks the necessary activity against L. monocytogenes and would require the addition of ampicillin. Things to remember:Empiric antibiotic treatment for neonatal meningitis should cover group B streptococcus, Escherichia coli, and Listeria monocytogenes. The preferred options are ampicillin plus either cefotaxime or gentamicin

A 57-year-old female comes to the emergency department due to fever, severe headache, stiff neck, and nausea. She reports the symptoms had a rapid onset 24 hours ago and have progressively worsened. Past Medical History: asthma, heartburn, seasonal allergies Allergies: sulfa (diffuse hives) Medications: Flovent, Proair HFA, loratadine, Pepcid AC Vital Signs: BP 115/75 mmHg, HR 100 bpm, RR 18 bpm, T 102°F (39°C), Ht 5′ 4″, Wt 60 kg Laboratory Tests:Hemoglobin 12.8 g/dLPlatelets 240,000 cells/mm3White blood cells 15,000 cells/mm3Sodium 136 mEq/LPotassium 4.2 mEq/LChloride 102 mEq/LBicarbonate 12 mEq/LBlood urea nitrogen 25 mg/dLCreatinine 1.2 mg/dLCalcium 8.4 mg/dLGlucose 100 mg/dL Diagnostic Tests:Lumbar puncture: high opening pressure, cerebrospinal fluid analysis pending Which of the following antibiotic regimens should be ordered empirically?

A. Ampicillin + ceftriaxone + vancomycin Fever, headache, neck stiffness, and nausea, as well as a high opening pressure during a lumbar puncture, are consistent with acute bacterial meningitis. Most cases are caused by Streptococcus pneumoniae or Neisseria meningitidis, but Listeriamonocytogenes is also a possible pathogen in patients who are age > 50 or immunocompromised. Urgent empiric treatment is necessary to limit neurologic deficits and reduce mortality. First-line antibiotic therapy targets the most common pathogens and includes a third-generation cephalosporin (eg, ceftriaxone) and vancomycin (for dual activity against potential cephalosporin-resistant S. pneumoniae). Ampicillin is added to the regimen to cover Listeria based on risk. These agents have better penetration of the cerebrospinal fluid (CSF) when the meninges are inflamed. (Choice B) Gentamicin does not adequately penetrate the CSF of adults to treat meningitis because the blood-brain barrier (BBB) is fully developed. Gentamicin is an option in neonates because the BBB is still developing, which allows higher serum concentrations to cross into the CSF. (Choice C) Cefotaxime alone does not cover Listeria or cephalosporin-resistant S. pneumoniae, both of which are of concern due to this patient's age. (Choice D) Because this patient is over age 50, ampicillin must be added to ceftriaxone and vancomycin to cover Listeria. (Choice E) Because community-acquired bacterial meningitis is not commonly caused by Pseudomonas, piperacillin/tazobactam is too broad. In addition, it does not cover Listeria and has poor penetration into the CSF compared to other recommended agents. Pseudomonal activity should be added (using cefepime or meropenem) when meningitis is secondary to a penetrating head trauma or post neurosurgery. Things to remember:Adults with suspected acute bacterial meningitis require urgent empiric treatment with ceftriaxone and vancomycin. Ampicillin should be added in patients age > 50 to provide empiric activity against Listeria.

RD is a 56-year-old female who comes to the emergency department because of an ulcer on the sole of her left foot. The ulcer has been present for a few weeks but has now started draining pus. Other symptoms include fever and malaise. The patient missed her last appointment with her primary healthcare provider and admits to medication nonadherence. Past Medical History: type 2 diabetes mellitus, peripheral neuropathy, obesity, hypertension, dyslipidemia Allergies: NKDA Home Medications:Metformin 1,000 mg PO BIDGlipizide 10 mg PO dailyDuloxetine 60 mg PO dailyLisinopril 20 mg PO dailyAtorvastatin 40 mg PO daily Vital Signs: BP 132/86 mmHg, HR 88 bpm, RR 16 bpm, T 101.2°F (38.4°C), Ht 5′ 4″, Wt 88 kg Physical Exam: left plantar foot ulcer with slightly malodorous purulent discharge, surrounding erythema, and edema; no evidence of bone involvement Laboratory Tests:White blood cells 13,500 cells/mm3Serum creatinine 1.2 mg/dLBlood urea nitrogen 18 mg/dLGlucose 228 mg/dLHemoglobin A1C 10.2%MRSA nasal swab negative Diagnostic Tests:MRI of the left foot: negative for osteomyelitis Additional Information:1. Admit to general medicine unit.2. Consult surgery for debridement.3. Obtain blood and wound cultures.4. Start IV antibiotics for diabetic foot infection. Which antibiotic is most appropriate to empirically treat this patient's infection?

A. Ampicillin/ Sulbactam Patients with diabetes mellitus have a high lifetime risk of developing foot ulcers. Major contributing factors include loss of sensation caused by peripheral neuropathy and inadequate wound healing resulting from hyperglycemia. The presence of erythema, edema, purulent discharge, fever, and leukocytosis indicate that this patient's ulcer has progressed to an infection. Diabetic foot infections are often polymicrobial, requiring broad-spectrum antibiotics that target gram-positive (eg, streptococci, staphylococci), gram-negative (eg, Enterobacteriaceae), and anaerobic (eg, Bacteroides spp.) pathogens. Select options include ampicillin/sulbactam, piperacillin/tazobactam, a carbapenem (eg, ertapenem, meropenem), or metronidazole plus a cephalosporin (eg, ceftriaxone) or fluoroquinolone (eg, levofloxacin). Empiric treatment for methicillin-resistant Staphylococcus aureus (MRSA) or Pseudomonas aeruginosa is not routinely recommended unless risk factors (eg, colonization, prior infection) are present. (Choice B) Azithromycin (a macrolide) is not recommended due to increased bacterial resistance rates; it also has limited gram-negative activity (does not cover enteric gram-negative rods) and lacks anaerobic activity. (Choice C) Ciprofloxacin (a fluoroquinolone) targets primarily gram-negative pathogens, including P. aeruginosa, and does not have adequate activity against gram-positive or anaerobic bacteria. (Choice D) Penicillin G aqueous has no appreciable activity against S. aureus, gram-negative bacteria, or gram-negative anaerobes. (Choice E) Vancomycin monotherapy lacks activity against gram-negative and anaerobic pathogens. Vancomycin can be added to the empiric treatment regimen in patients with MRSA risk. Because this patient has no prior MRSA infection and a negative MRSA nasal swab, vancomycin is not needed. Things to remember:Diabetic foot infections are typically polymicrobial (ie, caused by gram-positive, gram-negative, and anaerobic pathogens). Patients with moderate to severe infection (eg, erythema, purulence, fever, leukocytosis) are initially treated with broad-spectrum parenteral antibiotics such as ampicillin/sulbactam, piperacillin/tazobactam, or metronidazole plus

MS is a 62-year-old male who was diagnosed with hospital-acquired pneumonia (HAP) 3 days ago. He has no known drug allergies. A respiratory sample was collected prior to initiating empiric antibiotic treatment with cefepime, levofloxacin, and vancomycin. The patient's symptoms have since improved and the medical team is ready to de-escalate antibiotic therapy based on the final culture and susceptibility results shown below. Which antibiotic regimen is most appropriate for continued treatment of HAP in this patient? Sputum culture Antibiotic tested Enterobacter cloacae Aztreonam R Cefepime S Ceftriaxone R Ciprofloxacin R Gentamicin S Levofloxacin R Meropenem S Piperacillin/tazobactam S Tobramycin S R = resistant; S = susceptible.

A. Cefepime Initial empiric antibiotic treatment for hospital-acquired pneumonia requires broad-spectrum activity against gram-negative bacilli (eg, Pseudomonas aeruginosa, Enterobacter cloacae) and Staphylococcus aureus. Resistant pathogens (eg, multidrug-resistant gram-negative bacilli, MRSA) should be targeted in patients with risk factors (eg, IV antibiotic use in the past 90 days). Culture and susceptibility of blood and respiratory samples (eg, induced sputum) is recommended to guide antibiotic de-escalation. The goal of de-escalation is to limit undesirable outcomes (eg, Clostridioides difficile infection, adverse effects, antibiotic resistance) by selecting the narrowest-spectrum regimen that targets the isolated pathogen(s). In this patient with noted clinical improvement, de-escalation to a single antibiotic is appropriate because one pathogen, E. cloacae, was isolated from the sputum culture. Cefepime is the best option based on the reported susceptibilities. (Choices B and C) The culture results indicate that the E. cloacae isolate is resistant to both ceftriaxone and levofloxacin. (Choice D) The E. cloacae isolate is reported as susceptible to both meropenem and tobramycin, but there is no clinical benefit to continuing "double coverage" for a single organism, and unnecessary adverse effects could occur (eg, nephrotoxicity with tobramycin). (Choice E) Although the E. cloacae isolate is susceptible to piperacillin/tazobactam, it has a broader spectrum of activity (eg, anaerobic coverage) than cefepime. In addition, with no objective evidence to support continuing vancomycin (eg, S. aureus was not isolated from the culture), the risks (eg, nephrotoxicity, antibiotic resistance) outweigh the benefits. Things to remember:Initial empiric antibiotic treatment for hospital-acquired pneumonia should be de-escalated based on culture and susceptibility results. The narrowest-spectrum antibiotic(s) to which the isolated pathogen(s) are reported as susceptible should be selected to limit adverse outcomes (eg, antibiotic resistance, adverse effects).

TT, a 54-year-old female with severe colitis secondary to ulcerative colitis, is being prepped for an elective partial colectomy. She has no known drug allergies. Which of the following preoperative antibiotic regimens provide an appropriate spectrum of activity to prevent a surgical site infection following this procedure? (Select ALL that apply)

A. Ampicillin/sulbactam C. Cefazolin plus metronidazole D. Cefoxitin Preoperative antibiotics, when appropriately selected and timed prior to the procedure, significantly reduce the incidence of surgical site infections (SSIs). The agent(s) with the narrowest spectrum that targets the bacterial flora at the surgical incision site(s) should be chosen. For procedures involving the gastrointestinal lumen (eg, colectomy), antibiotics that have activity against gram-negative and anaerobic bacteria in addition to normal skin flora (eg, Staphylococcus aureus, coagulase-negative staphylococci) are needed. Example regimens include: Ampicillin/sulbactam: The beta-lactamase inhibitor sulbactam extends ampicillin's limited spectrum to include more gram-positive [eg, methicillin-susceptible S. aureus (MSSA)], gram-negative, and anaerobic (eg, Bacteroides fragilis) activity (Choice A). Cefazolin plus metronidazole: Cefazolin, a first-generation cephalosporin, lacks activity against gram-negative anaerobes and must be combined with metronidazole to provide this necessary coverage (Choice C). Cefoxitin or cefotetan: Both agents are second-generation cephalosporins with similar gram-positive and expanded gram-negative activity compared to first-generation cephalosporins (eg, cefazolin). In addition, they both have activity against gram-negative anaerobes, which makes them unique (Choice D). (Choice B) Because cefazolin lacks activity against anaerobic bacteria, monotherapy is not appropriate to prevent SSIs following gastrointestinal surgeries. Cefazolin has activity against MSSA and streptococci, which are the primary organisms of concern for cardiac and orthopedic surgical procedures. (Choice E) Clindamycin provides activity against skin flora (eg, staphylococci, streptococci) and gram-positive anaerobes but lacks activity against aerobic and anaerobic gram-negative bacteria. Things to remember:Surgeries that require an incision into the gastrointestinal lumen require perioperative antibiotics with activity against gram-positive, gram-negative, and anaerobic bacteria. Example regimens include ampicillin/sulbactam, second-generation cephalosporins (cefoxitin or cefotetan), or cefazolin plus metronidazole.

OP, a 72-year-old female, is transported to the emergency department via ambulance after her son found her at home confused, feverish, and lethargic. The son states that the day prior, OP reported a cough, shortness of breath, and chills, which prompted him to check on her. In the emergency department, the patient is intubated and placed on a mechanical ventilator. Transfer to the intensive care unit is pending. OP has no known risk factors for MRSA or Pseudomonas aeruginosa. Past Medical History: chronic obstructive pulmonary disease, hypothyroidism, osteoporosis Allergies: no known drug allergies Vital Signs: BP 85/60 mmHg, HR 115 bpm, RR 33 bpm, O2 sat 81% on room air, T 102°F (38.9°C) Diagnostic Tests: Chest x-ray: left lower lobe infiltrate Which of the following antibiotic orders should the ICU pharmacist anticipate dispensing for this patient?

A. Ampicllin/ sulbactam + levofloxacin Guideline-recommended antibiotics for the inpatient treatment of community-acquired pneumonia (CAP) are based on the setting of care (intensive care unit vs. general medicine unit) and target the common CAP pathogens (eg, Streptococcus pneumoniae, Haemophilus influenzae, atypicals). For patients with severe CAP, the recommended treatment is a beta-lactam (eg, ampicillin/sulbactam) in combination with either a macrolide (eg, azithromycin) or a respiratory fluoroquinolone (eg, levofloxacin). These combination regimens provide double coverage of resistant S. pneumoniae and H. influenzae, and the macrolide or fluoroquinolone targets atypical pathogens. (Choice B) Ceftriaxone (a third-generation cephalosporin) covers S. pneumoniae and gram-negative rods but not atypicals. It should be paired with a macrolide or respiratory fluoroquinolone when treating severe CAP. Vancomycin (a glycopeptide) is recommended only for CAP treatment when methicillin-resistant Staphylococcus aureus (MRSA) risk factors are present (eg, prior respiratory isolation). (Choice C) Clarithromycin (a macrolide) has excellent activity against atypical pathogens; however, due to increasing resistance against S. pneumoniae, it should be paired with a beta-lactam when treating inpatient CAP. (Choice D) In patients admitted to the intensive care unit with severe CAP, a beta-lactam should be used in combination with the respiratory fluoroquinolone (eg, levofloxacin) due to the severity of illness and risk for treatment failure. Fluoroquinolone monotherapy has not been adequately studied in severe CAP. (Choice E) Piperacillin/tazobactam (a beta-lactam/beta-lactamase inhibitor combination) has broad-spectrum activity against gram-positive [eg, methicillin-susceptible S. aureus (MSSA)], gram-negative (eg, Enterobacter, Pseudomonas) and anaerobic (eg, Bacteroides fragilis) organisms but not atypicals. Piperacillin/tazobactam is an alternative treatment for inpatient CAP when the patient has risk factors for Pseudomonas (eg, prior respiratory isolation). Things to remember:Severe inpatient community-acquired pneumonia is treated with a beta-lactam plus either a macrolide or a respiratory fluoroquinolone.

A 25-year-old female comes to the urgent care clinic with a dry, irritable cough. She reports that her illness started 12 days ago with fatigue, runny nose, and a dry cough. Although her other symptoms have improved, the cough has worsened, becoming "hacking" in nature and more frequent, with some episodes lasting 1-2 minutes. While examining the patient, the physician notes an inspiratory "whoop" after the cough. A sputum PCR is positive for Bordetella pertussis. The patient has an allergy to sulfa (hives). Which treatment is preferred?

A. Azithromycin Pertussis, commonly known as whooping cough, is an acute respiratory illness caused by the gram-negative coccobacillus Bordetella pertussis. The illness begins with nonspecific respiratory symptoms (eg, rhinorrhea, generalized malaise, cough) that persist and worsen to a characteristic cough, generally described as a series of severe, forceful coughs followed by a vigorous inspiration that causes the distinctive "whoop" sound. Post-tussive emesis is also common. Diagnosis is confirmed with a nasopharynx swab culture or polymerase chain reaction (PCR) test. Because pertussis is highly contagious and associated with a higher risk of morbidity and mortality in infants and young children, antibiotics are recommended to prevent further spread of the disease. The first-line antibiotics, macrolides (azithromycin or clarithromycin), are effective at eradicating B. pertussis from infected patients. (Choice B) Clindamycin is active against gram-positive aerobes (eg, streptococci, staphylococci) and anaerobes, but it has no appreciable activity against gram-negative organisms, including B. pertussis. (Choice C) Linezolid is active only against gram-positive bacteria (including methicillin-resistant Staphylococcus aureus); therefore, it cannot treat pertussis. (Choice D) Because the patient has an allergy to sulfa, sulfamethoxazole/trimethoprim should be avoided. It would be an acceptable alternative for pertussis in patients without a sulfa allergy who cannot safely use macrolides. (Choice E) Antibiotic treatment eradicates B. pertussis from infected individuals and reduces transmission to others. Empiric treatment is recommended in any individual with clinical symptoms or microbiologically confirmed disease. Things to remember:Pertussis (whooping cough) is a respiratory illness caused by the gram-negative bacteria Bordetella pertussis. Antibiotic treatment with macrolides (azithromycin or clarithromycin) is recommended to prevent the spread of this highly contagious infection.

GS, a 68-year-old male, was recently admitted to the general medical unit of a hospital with a bleeding gastric ulcer caused by excessive use of NSAIDs. Three days after admission, he develops a fever and crackles are noted on pulmonary examination. Past Medical History: diverticulosis, osteoarthritis, peripheral artery disease, MRSA skin abscess (4 months ago) Allergies: penicillin (anaphylaxis) Medications:Protonix 40 mg PO BIDAcetaminophen 325 mg, 1-2 tablets PO Q6H PRN Vital Signs: BP 134/88 mmHg, HR 88 bpm, RR 22 bpm, O2 sat 90% on room air, T 101.7°F (38.7°C) Laboratory Tests:White blood cells 14,200 cells/mm3Hemoglobin 13.2 g/dLPlatelets 280,000 cells/mm3Serum creatinine 0.9 mg/dL Diagnostic Tests:Chest x-ray: right lower lobe infiltrateInduced sputum sample: Gram stain and culture pendingNasal swab: positive for MRSA Which empiric antibiotic regimen is most appropriate for this patient?

A. Aztreonam + vancomycin Hospital-acquired pneumonia (onset ≥ 48 hours after hospital admission) is commonly caused by nosocomial pathogens [eg, Pseudomonas aeruginosa, methicillin-resistant Staphylococcus aureus (MRSA)] and can be polymicrobial. Empiric broad-spectrum antibiotics should be initiated promptly with at least one drug active against P. aeruginosa and methicillin-susceptible S. aureus; additional agents may be necessary if there are risks for MRSA or multidrug-resistant (MDR) gram-negative pathogens. In this patient with a severe penicillin allergy, the best antipseudomonal treatments are aztreonam or a fluoroquinolone (eg, levofloxacin). The past MRSA infection and positive MRSA nasal swab (indicating colonization) warrant the addition of vancomycin or linezolid. (Choices B and C) Beta-lactam antibiotics (eg, ceftaroline, meropenem) should be avoided in patients with a severe penicillin allergy due to the risk of cross-reactivity. In addition, neither ceftaroline nor azithromycin has activity against P. aeruginosa; this combination is more appropriate for the treatment of inpatient community-acquired pneumonia. When there are no risks for MDR gram-negative pathogens (eg, IV antibiotic use in the past 90 days), there are no benefits to using dual antipseudomonal treatment (eg, meropenem + ciprofloxacin). (Choices D and E) Moxifloxacin (a respiratory fluoroquinolone) is not active against P. aeruginosa or MRSA. Linezolid can be used for MRSA but should be paired with a more effective antipseudomonal agent; tobramycin has activity against P. aeruginosa but monotherapy is not recommended due to poor lung penetration and increased toxicity (eg, nephrotoxicity, ototoxicity) from the high doses needed. Things to remember:The preferred treatment regimen for hospital-acquired pneumonia in patients with a severe penicillin allergy and MRSA risks is aztreonam or levofloxacin, to cover Pseudomonas aeruginosa, plus vancomycin or linezolid, to cover MRSA.

A 67-year-old female is scheduled for an elective total knee replacement surgery. She has a history of hypertension, osteoarthritis, and dyslipidemia. Preoperative vital signs and laboratory tests are within normal limits, and the patient has no known drug allergies. Which of the following intravenous antibiotics should be administered prior to surgery?

A. Cefazolin Preoperative antibiotics reduce the risk of surgical site infections (SSIs) and subsequent morbidity and mortality. Narrow-spectrum antibiotics should be administered to target the most likely bacteria that can contaminate the surgical site. For most procedures, including orthopedic surgeries, skin flora (eg, Staphylococcus aureus, coagulase-negative staphylococci, streptococci) is the biggest concern, and the treatment of choice is intravenous cefazolin. Surgeries that require entry into a visceral space (eg, the abdomen) require broader-spectrum antibiotics active against gram-negative rods and anaerobes. The timing of antibiotic administration is also critical to ensure adequate tissue concentrations before the first incision and throughout the duration of the surgery. (Choice B) The combination of intravenous cefazolin and metronidazole would be appropriate if anaerobes are part of the local bacterial flora at the surgical site (eg, appendectomy, colon resection). (Choice C) Ciprofloxacin does not have adequate activity against gram-positive pathogens (eg, staphylococci) and, with extensive gram-negative activity, it is too broad-spectrum for orthopedic surgery prophylaxis. Ciprofloxacin is an appropriate prophylactic antibiotic for genitourinary surgeries (eg, cystoscopy). (Choice D) Clindamycin has activity against gram-positive aerobes (eg, staphylococci, streptococci) and anaerobes. Its use in SSI prevention is reserved for when cefazolin cannot be used (eg, severe penicillin or cephalosporin allergies). (Choice E) Although ertapenem is effective at preventing SSIs when polymicrobial coverage is needed (eg, gastrointestinal surgeries), it is generally not recommended due to concerns of promoting resistance (eg, carbapenem-resistant Enterobacteriaceae). Things to remember:Cefazolin is the preferred antibiotic to prevent surgical site infections caused by skin flora (staphylococci, streptococci). For surgeries involving visceral spaces (eg, the intestinal tract), an antibiotic regimen with gram-negative and anaerobic activity is required (eg, cefazolin plus metronidazole).

TT is a 54-year-old male admitted to a hospital general medical unit with community-acquired pneumonia. He has a history of poorly controlled asthma resulting in frequent hospitalizations. His last hospitalization was 5 months ago due to acute respiratory failure that required mechanical ventilation; respiratory cultures at that time identified Pseudomonas aeruginosa (see susceptibility results below). An MRSA nasal swab performed in the emergency department today is negative. He has no known drug allergies. Bronchoalveolar lavage culture results (5 months ago)Antibiotic testedPseudomonas aeruginosaAmikacinSCefepimeSCeftazidimeSCiprofloxacinRGentamicinRLevofloxacinRMeropenemSPiperacillin/tazobactamRTobramycinSR = resistant; S = susceptible. Based on this information, which empiric antibiotic regimen is most appropriate for the treatment of this patient?

A. Cefepime + azithromycin Patients with community-acquired pneumonia (CAP) and prior respiratory isolation of Pseudomonas should receive empiric antibiotics targeted at Pseudomonas and the usual CAP pathogens. Antipseudomonal antibiotics include cefepime, ceftazidime, piperacillin/tazobactam, meropenem, and aztreonam (if severely allergic to penicillin). Previous culture and susceptibility results can also help guide empiric therapy. If methicillin-resistant Staphylococcus aureus (MRSA) risk is present (eg, positive MRSA nasal swab, prior respiratory isolation), vancomycin or linezolid should be included in the antibiotic regimen. (Choice B) Ceftaroline (a fifth-generation cephalosporin) has gram-positive activity, including MRSA for skin and soft tissue infections, and similar gram-negative activity as ceftriaxone but no activity against Pseudomonas. When used for CAP, it must be combined with an antibiotic that covers atypical organisms (eg, a macrolide). (Choice C) Ceftriaxone does not have activity against Pseudomonas. Although levofloxacin has activity against this organism, it would not be preferred empirically over an antipseudomonal beta-lactam. In addition, the previous Pseudomonas culture indicates fluoroquinolone resistance. (Choice D) Ertapenem (a carbapenem) lacks activity against Pseudomonas, Acinetobacter, Enterococcus, and atypical organisms. It is often reserved for polymicrobial infections (due to gram-negative, gram-positive, and anaerobic bacteria) and resistant organisms that express an extended-spectrum beta-lactamase (ESBL). (Choice E) Although piperacillin/tazobactam can be expected to have activity against Pseudomonas, there is evidence of resistance on a previous culture. Tobramycin (an aminoglycoside) has activity against Pseudomonas. However, neither piperacillin/tazobactam nor tobramycin has activity against atypical organisms. Things to remember:Patients with community-acquired pneumonia and risk factors for MRSA and/or Pseudomonas (eg, prior isolation on a respiratory sample, hospitalization in the past 90 days with intravenous antibiotic exposure) should receive empiric antibiotics with activity against these resistant organisms and atypical pathogens.

MR is a 55-year-old female who comes to the emergency room with right upper quadrant abdominal pain, a temperature of 102.9°F (39.4°C), chills, and jaundiced skin. She has a past medical history of hypertension, obesity, and hypertriglyceridemia. Notable laboratory findings include an elevated white blood cell count and hyperbilirubinemia. A right upper quadrant ultrasound reveals common bile duct dilation and multiple gallbladder stones. The patient is diagnosed with acute cholangitis. Medication orders for IV fluids and broad-spectrum antibiotics are sent to the pharmacy. The patient has no known drug allergies. Which antibiotic regimen is the best option to empirically treat this patient's infection?

A. Ceftriaxone plus metronidazole Intra-abdominal infections (eg, acute cholangitis, appendicitis, diverticulitis) occur when intestinal mucosal barriers are disrupted by an inflammatory process, obstruction, or perforation. This allows bowel flora to contaminate the abdominal space, resulting in a polymicrobial infection that manifests with symptoms of abdominal pain, fever, and an elevated white blood cell count. Regardless of the specific type of infection, empiric broad-spectrum antibiotics with activity against gram-positive (eg, streptococci), gram-negative (eg, Escherichia coli, Klebsiella spp., Enterobacter spp.), and anaerobic (eg, Bacteroides fragilis) pathogens are indicated. Treatment options for community-acquired infections include metronidazole plus a cephalosporin (eg, ceftriaxone, cefotaxime) or fluoroquinolone (eg, levofloxacin). Patients with severe or nosocomial infections should receive a regimen with activity against resistant pathogens, including Pseudomonas aeruginosa (eg, piperacillin/tazobactam, metronidazole plus cefepime, meropenem). (Choice B) Clindamycin has activity against gram-positive aerobes (eg, streptococci, staphylococci) and anaerobes but lacks the necessary activity against gram-negative pathogens. (Choices C and D) Both levofloxacin and sulfamethoxazole/trimethoprim lack activity against anaerobic organisms, making monotherapy with either antibiotic inappropriate for empiric treatment of intra-abdominal infections. (Choice E) When used alone, tobramycin (an aminoglycoside) has activity against only gram-negative pathogens (not gram-positive or anaerobic organisms). Because of their associated toxicities (eg, nephrotoxicity), aminoglycosides are typically reserved for intra-abdominal infections that require double coverage of multidrug-resistant gram-negative pathogens. Things to remember:Intra-abdominal infections (eg, acute cholangitis) require broad-spectrum empiric antibiotic treatment to target gram-positive, gram-negative, and anaerobic organisms. For community-acquired infections (eg, no risk for Pseudomonas aeruginosa), common regimens include metronidazole in combination with a cephalosporin (eg, ceftriaxone) or fluoroquinolone.

HP is a 30-year-old, 15-week pregnant female who comes to her obstetrician for a regularly scheduled prenatal visit. A urine sample collected and sent for culture shows 106 CFU/mL of Escherichia coli, susceptible to all tested antibiotics. She has no urinary symptoms and "feels good." Her vital signs are normal, and she has no known drug allergies. Which treatment is recommended for this patient?

A. Cephalexin Asymptomatic bacteriuria (ASB) is defined as ≥ 105 CFU/mL of bacteria on urine culture and the absence of clinical symptoms of a urinary tract infection (eg, dysuria, suprapubic tenderness). In nonpregnant patients, treatment of ASB with antibiotics is not recommended. However, untreated ASB during pregnancy increases risks, including progression to pyelonephritis, preterm birth, and/or low newborn birth weight. Therefore, pregnant women are routinely screened for bacteriuria and treated with antibiotics, if positive. Escherichia coli is the most common pathogen that causes ASB, and selection of an antibiotic is based on the culture and susceptibility report. Beta-lactams (eg, cephalexin, amoxicillin/clavulanate) are preferred for ASB treatment during pregnancy; fosfomycin, nitrofurantoin, or sulfamethoxazole/trimethoprim are alternatives (eg, beta-lactam allergy, resistance). Nitrofurantoin and sulfamethoxazole/trimethoprim should only be used if other options are not available because of a greater potential risk to the fetus. (Choices B and C) Ciprofloxacin and doxycycline should be avoided during pregnancy because of risks to the fetus (eg, musculoskeletal abnormalities) and the availability of safer alternatives. (Choice D) Penicillin V potassium has activity against streptococci and enterococci but lacks intrinsic activity against E. coli. (Choice E) Antibiotic treatment is recommended for pregnant patients with asymptomatic bacteriuria to prevent the progression to a urinary tract infection and reduce the risk of low birth weight and preterm birth. Things to remember:Screening for asymptomatic bacteriuria is routine during pregnancy. If identified, antibiotic treatment is recommended to prevent risks (eg, preterm birth); the preferred antibiotics are beta-lactams (eg, cephalexin, amoxicillin/clavulanate). Fosfomycin, nitrofurantoin, and sulfamethoxazole/trimethoprim are alternatives if a beta-lactam allergy or resistance is present.

Empiric antibiotics and steroids have been ordered for a 45-year-old patient with suspected community-acquired bacterial meningitis. Cultures and a lumbar puncture are pending. If antibiotics are expected to be administered at 1400, which of the following is correct regarding the selection and timing of steroids in this patient?

A. Dexamethasone, administer at 1400 Dexamethasone reduces neurological complications (eg, hearing loss) and mortality from Streptococcus pneumoniae meningitis when administered 15-20 minutes before or at the same time as the first dose of empiric intravenous antibiotics. Because the causative organism is usually not known when treatment is initiated and given that S. pneumoniae is the most common pathogen, dexamethasone should be administered in all cases of community-acquired meningitis until diagnostic tests (eg, cultures) identify an organism. If an organism other than S. pneumoniae is identified, dexamethasone should be discontinued. The recommended dose of dexamethasone is 0.15 mg/kg intravenously every 6 hours; in confirmed cases of S. pneumoniae, it should be continued for 4 days. (Choice B) Dexamethasone administration after the first dose of IV antibiotics has not shown clinical benefit. Early administration alleviates the inflammatory changes and subsequent neurological deficits caused by bacterial meningitis. (Choices C and D) Prednisone is an oral glucocorticoid; the time it takes for absorption and distribution into the cerebrospinal fluid (CSF) would be too long to exert any beneficial effect. (Choice E) Although antibiotics rely on the inflamed meninges to enter the CSF, the severity of the inflammation is life-threatening and overrules any theoretical concerns about reduced antibiotic penetration resulting from the anti-inflammatory properties of steroids. Furthermore, higher doses of antibiotics are used for meningitis to ensure distribution into the CSF. Things to remember:Dexamethasone, administered 15-20 minutes before or at the same time as the first dose of IV antibiotics, is recommended to reduce neurological complications (eg, hearing loss) and mortality in bacterial meningitis cases caused by Streptococcus pneumoniae.

Which of the following adverse effects are associated with isoniazid? (Select ALL that apply)

A. Hepatoxicity D. Lupus-like syndrome E. Peripheral neuropathy Isoniazid (INH) is used in treatment regimens for both latent and active tuberculosis. Adverse effects of all commonly used antitubercular medications include gastrointestinal upset and hepatotoxicity; the latter requires monitoring of liver function tests [eg, transaminases (ALT/AST)] at baseline and periodically (eg, monthly) while on therapy (Choice A). Additional adverse effects specific to INH include: Drug-induced lupus erythematosus: An autoimmune response caused by exposure to a drug. The most common symptoms include a characteristic rash on sun-exposed areas (eg, "butterfly rash" on the face), fever, arthralgia, and myalgia. Initial management includes discontinuing the offending medication (Choice D). Peripheral neuropathy: Vitamin B6 (pyridoxine) acts as a cofactor in the synthesis of neurotransmitters. The activity of pyridoxine is inhibited by metabolites of INH, resulting in peripheral neuropathy, ataxia, and paresthesia. Concomitant administration of pyridoxine with INH reduces the risk of these neurologic adverse effects (Choice E). (Choice B) Hyperuricemia (elevated serum uric acid levels) can occur with the antitubercular drug pyrazinamide, which is contraindicated during an acute gout attack. Other medications that can cause elevated uric acid include niacin, calcineurin inhibitors (cyclosporine and tacrolimus), and diuretics (loop and thiazide). (Choice C) Drugs known to cause hypokalemia (potassium < 3.5 mEq/L) include albuterol, amphotericin B, loop diuretics (eg, furosemide), and insulin. Things to remember:Isoniazid is used to treat latent and active tuberculosis and has important adverse effects, including hepatotoxicity (monitor liver function tests), peripheral neuropathy [prevent with coadministration of vitamin B6 (pyridoxine)], and drug-induced lupus erythematosus (requires drug discontinuation).

A 54-year-old male is diagnosed with active pulmonary tuberculosis and hospitalized in the internal medicine unit. Which of the following strategies are recommended to prevent transmission of tuberculosis? (Select ALL that apply)

A. Isolate the patient in a negative pressure room C. Place that patient in a single-occupancy room D. Wear a fitted N95 respirator mask when inside the patient's room Tuberculosis (TB) is caused by the bacteria Mycobacterium tuberculosis. The active phase of pulmonary TB is highly transmissible by airborne particles (droplets) that are generated when a patient coughs, sneezes, speaks, or even sings. The droplets can remain suspended in the air for prolonged periods, allowing them to spread throughout a room or building. Infection occurs when a susceptible person inhales droplets containing M. tuberculosis. The following strategies are recommended to prevent the spread of M. tuberculosis in healthcare settings: Isolate the patient in a single-occupancy, negative pressure room such that the air flow is directed into the patient's room when the door is opened, thereby preventing any respiratory droplets suspended in the air from escaping into adjacent areas (Choices A and C). Healthcare workers and visitors should wear fitted N95 respirator masks or powered air-purifying respirators when inside the patient's room (Choice D). (Choice B) A positive pressure room forces air to flow out of the patient's room when the door is opened and would allow infectious droplets that are suspended in the air to spread to other areas. Positive pressure rooms are sometimes used for neutropenic patients to prevent the spread of potentially infectious air into the patient's room from adjacent areas. (Choice E) Because TB is not spread through direct contact, gowns are not recommended inside the patient's room. Things to remember:Active pulmonary tuberculosis is highly contagious through respiratory droplets that are expelled when an infected patient coughs, sneezes, speaks, or sings. The spread of infectious droplets can be prevented in healthcare settings by placing patients in single-occupancy, negative pressure rooms and by wearing fitted N95 respirator masks.

LG is a 47-year-old male who is hospitalized with a Clostridioides difficile infection. The internal medicine pharmacist intends to enter the patient's room to perform medication reconciliation and counsel the patient. Which of the following actions should the pharmacist perform when caring for this patient? (Select ALL that apply)

A. Review the patient's profile for unnecessary antibiotics B. Review the patient's profile for unnecessary proton pump inhibitors C. Wash h ands with soap and water after leaving the patient's room E. Wear a gown and gloves while inside the patient's room Clostridioides difficile is highly transmissible via the fecal-oral route through the ingestion of spores. Patients infected or colonized with C. difficile shed spores into the surrounding environment that contaminate the hands and clothing of healthcare workers and subsequently spread to other patients. The following measures help prevent the spread of C. difficile: Hand hygiene: Handwashing with soap and water is more effective at removing C. difficile spores than alcohol-based hand sanitizers (Choice C). Contact precautions: Gowns and gloves should be worn when inside the patient's room to prevent contamination of clothes and hands (Choice E). Isolation: Patients should be isolated in a single-occupancy room with a dedicated bathroom. Medication profile review: Antibiotic exposure is the major risk factor for developing C. difficile infection (CDI) and excessive use of proton pump inhibitors has been associated with increased risk for CDI. Pharmacists should review patient profiles for appropriate use of these medications based on local and national guidelines and discuss recommendations to improve their use with prescribers. Continuing unnecessary antibiotics can prolong and/or worsen an active case of CDI (Choices A and B). (Choice D) Masks are indicated for airborne diseases (eg, influenza) and do not provide additional protection against C. difficile. Hair covers are used in sterile compounding to prevent contamination of the prepared product. Things to remember:The spread of Clostridioides difficile in the hospital can be prevented by instituting contact precautions and handwashing with soap and water when caring for patients suspected or confirmed to have the disease. Antibiotic stewardship and avoiding unnecessary proton pump inhibitors can help minimize patient risk of developing C. difficile infection.

A previously healthy 7-month-old child (weight: 20 lbs) is prescribed Augmentin for acute otitis media. The pharmacist will prepare a suspension containing 400 mg of amoxicillin and 57 mg of clavulanate per 5 mL. Which volume and frequency would provide the correct dose for this indication?

B. 5 mL twice daily High-dose amoxicillin is a first-line treatment for acute otitis media (AOM). The recommended pediatric dose is 90 mg/kg/day administered in two divided doses. When Augmentin (amoxicillin/clavulanate) is prescribed, the dose is based on the amoxicillin component and an oral formulation (eg, suspension, chewable tablet) appropriate for the age of the child should be selected. Using dimensional analysis, perform the following steps to calculate the correct dose: Calculate the recommended total daily dose of amoxicillin. 20 lbs×1 kg2.2 lbs×90 mgkg⋅day=818.181 mg of amoxicillin per day Convert the total daily dose to a volume (milliliters) using the preparation of 400 mg of amoxicillin per 5 mL. 818.181 mgday×5 mL400 mg=10.22 mL of amoxicillin per day Divide the amoxicillin daily dose in milliliters into 2 doses. 10.22 mLday×1 day2 doses=5.11 mL per dose The correct volume and frequency for this patient is 5 mL twice daily. (Choice A) A dose of 2.5 mL twice daily would provide 44 mg/kg/day of amoxicillin, which is too low to penetrate the middle ear fluid and obtain concentrations effective at treating AOM. (Choices C and E) Doses of 7.5 mL twice daily and 15 mL once daily would provide a total daily dose of 132 mg/kg/day of amoxicillin, which exceeds the recommended dose of 90 mg/kg/day. In addition, the short half-life of amoxicillin requires at least twice daily administration. (Choice D) A dose of 10 mL twice daily would provide 176 mg/kg/day of amoxicillin, which would result in a two-fold overdose to the patient. Things to remember:High-dose amoxicillin at a dose of 90 mg/kg/day in two divided doses is the first-line treatment for acute otitis media.

Medication orders for active pulmonary tuberculosis are sent to the pharmacy for a 27-year-old female. While processing the orders, the pharmacist receives an alert, warning of a drug interaction between rifampin and an existing medication previously prescribed to this patient. Which of the following medications would prompt an alert?

B. Apixaban Rifampin, an antibacterial used in latent and active tuberculosis treatment regimens, is a moderate or strong inducer of several cytochrome P450 (CYP450) isoenzymes (eg, CYP3A4, 2C9, 2C19). It also induces gastrointestinal P-glycoprotein (P-gp) efflux pumps, which transport drugs into the gastrointestinal lumen for fecal excretion, thereby decreasing their systemic absorption. Drug interactions between dual CYP3A4 and P-gp substrates and inducers can require an increase in the substrate drug dose or, in many cases, selection of alternate drugs to avoid the interaction altogether. Apixaban, a direct oral anticoagulant, is a substrate of both CYP3A4 and P-gp. When taken concurrently with rifampin, apixaban serum concentrations decrease significantly because of: Decreased absorption: More drug is eliminated via P-gp efflux. Increased metabolism: A higher amount of drug is converted via CYP3A4 to inactive compounds. The subsequent decreased anticoagulant effects can lead to serious outcomes, such as thromboembolism or stroke. For this reason, it is recommended to avoid concurrent use of apixaban with dual P-gp and strong CYP3A4 inducers. (Choices A, C, D, and E) Neither allopurinol (a xanthine oxidase inhibitor), chlorthalidone (a thiazide diuretic), nor ramipril (an angiotensin-converting enzyme inhibitor) functions as a substrate, inducer, or inhibitor of CYP450 isoenzymes or P-gp efflux pumps. Methotrexate (an antifolate metabolite) is a minor substrate of P-gp but has no CYP450 metabolism. There are no relevant interactions between these drugs and rifampin that would trigger a safety alert. Things to remember:Rifampin is a dual P-glycoprotein and strong CYP3A4 inducer. Concurrent use with apixaban (a substrate for both P-gp and CYP3A4) should be avoided because the interaction can result in significantly decreased apixaban serum concentrations and clinical efficacy.

GR is a 72-year-old female with a past medical history of type 2 diabetes mellitus, chronic kidney disease, hypertension, and dyslipidemia. She was admitted to the hospital 4 days ago with hyperosmolar hyperglycemic state, resulting from several missed days of insulin treatment. Today, she developed new-onset shortness of breath, fever, and an elevated white blood cell count. A chest x-ray reveals consolidations in the left lower lobe. She has not received intravenous antibiotics in the past 90 days and an MRSA nasal swab is negative. Which of the following antibiotics, when used as monotherapy, provide the desired spectrum of activity for this infection? (Select ALL that apply)

B. Cefepime D. Piperacillin/tazobactam Hospital-acquired pneumonia (HAP) has an onset > 48 hours after hospital admission. The causative pathogens can be antibiotic-resistant and include gram-negative rods (eg, Pseudomonas, Escherichia coli, Klebsiella pneumoniae) and gram-positive cocci (eg, Staphylococcus aureus). At a minimum, empiric treatment for HAP should consist of an antibiotic with activity against Pseudomonas and methicillin-susceptible S. aureus (MSSA), such as cefepime, piperacillin/tazobactam, meropenem, or imipenem/cilastatin (Choices B and D). This patient has no additional risk factors for antibiotic resistance [eg, IV antibiotic use in the past 90 days, recent respiratory isolation of methicillin-resistant S. aureus (MRSA), high prevalence of pathogen resistance in the hospital unit]; therefore, it is not necessary to add vancomycin or linezolid (for MRSA) or a second antipseudomonal antibiotic. (Choice A) Aztreonam (a monobactam) has activity only against gram-negative organisms and cannot be used as monotherapy for empiric HAP treatment because it lacks activity against MSSA. It may be used for HAP if combined with a gram-positive antibiotic (eg, vancomycin or linezolid). (Choice C) Ceftriaxone (a third-generation cephalosporin) has activity against gram-positive (eg, S. pneumoniae, MSSA) and gram-negative organisms (eg, E. coli, Klebsiella spp.) but lacks activity against Pseudomonas. (Choice E) Although aminoglycosides (eg, tobramycin) are active against Pseudomonas, monotherapy for HAP should be avoided because of poor lung penetration and the risk of toxicity from the high doses needed. In addition, tobramycin lacks sufficient activity against MSSA. Things to remember:Hospital-acquired pneumonia has an onset > 48 hours after hospital admission and should be treated with antibiotics that cover Pseudomonas and methicillin-susceptible Staphylococcus aureus (eg, cefepime, piperacillin/tazobactam). Additional antibiotics should be administered if risk factors for methicillin-resistant S. aureus or multidrug-resistant gram-negative rods are present.

PM is a 14-month-old male brought to the office by his mother. He was last seen 3 days ago for acute otitis media and was prescribed amoxicillin/clavulanate. The patient returns with persistent symptoms of an ear infection, and the mother is concerned because he keeps spitting out the medication. Which of the following medications can be given intramuscularly and empirically treats acute otitis media?

B. Ceftriaxone Acute otitis media (AOM) is commonly caused by Streptococcus pneumoniae, Haemophilus influenzae, or Moraxella catarrhalis. The first-line treatment of high-dose amoxicillin targets these organisms; treatment failure, defined as no improvement or worsening symptoms over 48-72 hours, can occur if the causative organism is resistant or if the patient does not tolerate the medication (eg, vomiting, consistently spitting out the medication). If palatability of an oral suspension is a barrier to tolerance, the addition of flavoring should be attempted to improve adherence. Intramuscular ceftriaxone administered once daily into a large muscle mass (such as the anterolateral thigh) for 1-3 days is a recommended alternative treatment for AOM in cases of treatment failure or when there is a history of a mild hypersensitivity reaction to penicillin. (Choices A and E) Azithromycin and vancomycin cannot be administered intramuscularly because they require reconstitution and dilution to small concentrations (eg, ≤ 5 mg/mL) with a large volume of diluent (eg, 250-500 mL). In general, intramuscular injections are limited to a total volume of 2-4 mL. (Choice C) Although gentamicin can be administered intramuscularly, it lacks activity against S. pneumoniae and cannot empirically treat AOM. (Choice D) Penicillin lacks activity against H. influenzae and M. catarrhalis. Penicillin G benzathine is administered intramuscularly and is used to treat streptococcal pharyngitis; it is also the treatment of choice for syphilis. Things to remember:Treatment failure in acute otitis media can result from resistance to first-line antibiotics or an inability to tolerate oral suspension formulations. Intramuscular ceftriaxone is a recommended alternative in such cases or in cases of a mild hypersensitivity reaction to penicillin.

JS is a 68-year-old male who arrives at the emergency department with new-onset confusion in the past 24 hours, as well as fever, chills, and cough with purulent sputum that began 2 days ago. JS has had no recent exposure to antibiotics or any recent hospitalizations. At this time, he is stable enough to be admitted to the internal medicine unit. Past Medical History: hypertension, depression, dyslipidemia Allergies: no known drug allergies Medications:Lisinopril 20 mg once dailyAtorvastatin 40 mg once dailySertraline 100 mg once daily Vital Signs: BP 105/70 mmHg, HR 110 bpm, RR 22 bpm, T 101.7°F (38.7°C) Diagnostic Tests:Chest x-ray: right lower lobe infiltrate Which of the following antibiotic regimens is appropriate to empirically treat community-acquired pneumonia in this patient?

B. Ceftriaxone + azithromycin Antibiotic selection for community-acquired pneumonia (CAP) in the inpatient setting is based on the most common organisms (eg, Streptococcus pneumoniae, atypical organisms) and disease severity. For patients with nonsevere CAP, a beta-lactam (eg, ceftriaxone) plus a macrolide (eg, azithromycin) or respiratory fluoroquinolone monotherapy (eg, levofloxacin, moxifloxacin) is recommended. The treatment of severe CAP consists of a beta-lactam combined with either a macrolide or a respiratory fluoroquinolone. (Choice A) Aztreonam (a monobactam) has activity against gram-negative bacteria only and is used primarily in patients with a severe penicillin allergy (due to a low risk of cross-reactivity) and risk factors for Pseudomonas. Vancomycin (a glycopeptide) is mostly used for methicillin-resistant staphylococcal infections. Both drugs lack activity against atypical pathogens. (Choice C) Ceftriaxone has activity against S. pneumoniae and gram-negative bacteria. When used for CAP, a second agent with activity against atypical pathogens must be added. Metronidazole only has activity against anaerobic bacteria (eg, Bacteroides fragilis) and does not cover atypical organisms. (Choice D) Ciprofloxacin (a fluoroquinolone) has poor activity against S. pneumoniae and for this reason is not recommended for the treatment of CAP. (Choice E) Meropenem (a carbapenem) is a broad-spectrum antibiotic with activity against gram-positive and gram-negative bacteria (including Pseudomonas) and anaerobes (eg, B. fragilis). It is generally reserved for multidrug resistant and/or polymicrobial infections. It lacks activity against atypical organisms and is not recommended for the treatment of CAP. Things to remember:The recommended treatment for hospitalized patients with nonsevere community-acquired pneumonia is a beta-lactam plus a macrolide or monotherapy with a respiratory fluoroquinolone. For severe disease, patients should receive a beta-lactam in combination with either a macrolide or a respiratory fluoroquinolone.

A 36-year-old female (weight: 60 kg, height: 62 inches) visits her gynecologist for an annual exam. She has no concerning symptoms but mentions having different sexual partners in the last year. Screening tests are positive for Neisseria gonorrhoeae and negative for Chlamydia trachomatis. The patient has no known drug allergies. Which treatment is most appropriate?

B. Ceftriaxone 500mg IM x 1 dose Gonorrhea is a sexually transmitted disease caused by Neisseria gonorrhoeae, an aerobic, gram-negative diplococcus. Monotherapy with a single dose of intramuscular ceftriaxone (500 mg if patient weight is less than 150 kg, or 1 gram if 150 kg or greater) is the recommended treatment if co-infection with chlamydia has been ruled out. These doses can treat gonococcal isolates with elevated minimum inhibitory concentrations to ceftriaxone, thereby preventing treatment failure and further development of resistance. When there is a documented or suspected co-infection with chlamydia, doxycycline 100 mg twice daily for 7 days should be added to the ceftriaxone regimen. (Choice A) Monotherapy with azithromycin is not recommended due to concerns of rising resistance in N. gonorrhoeae. Azithromycin 1 gram as a single dose is the treatment of choice for chlamydia when doxycycline is contraindicated (eg, pregnancy). (Choice C) Clindamycin has activity against streptococci, staphylococci, and anaerobes; it does not have any gram-negative activity and cannot treat N. gonorrhoeae. It is used as a treatment for bacterial vaginosis. (Choice D) N. gonorrhoeae is resistant to all tetracyclines, including doxycycline. Doxycycline 100 mg BID for 7 days is the preferred treatment for chlamydia in nonpregnant adults. (Choice E) Metronidazole does not have activity against N. gonorrhoeae or any aerobic organisms. Metronidazole only has an anaerobic spectrum of activity and is a recommended treatment for bacterial vaginosis and trichomoniasis. Things to remember:The recommended treatment for gonorrhea is a single dose of ceftriaxone 500 mg (if < 150 kg) or 1 gram (if ≥ 150 kg) intramuscularly. If chlamydial co-infection is suspected, doxycycline 100 mg BID for 7 days should be added.

JP is a 42-year-old male who comes to the urgent care clinic because of a "rash" on his right lower leg. He states that he first noticed an area of redness above his ankle approximately 4 days ago that has spread up the front of his shin. The patient reports no trauma to the area and has no fever, chills, or malaise. Past Medical History: obesity, seasonal allergies, gastroesophageal reflux disease Social History: smokes ~1 pack of cigarettes daily Allergies: NKDA Medications: cetirizine 10 mg PO QHS, famotidine 20 mg PO as needed for heartburn Vital Signs: BP 132/78 mmHg, HR 88 bpm, RR 14 bpm, T 100.1°F (37.8°C), Ht 5′ 9″, Wt 240 lbs Physical Exam: diffuse erythema of the right lower extremity with no clearly defined borders; the area is warm, tender to touch, and slightly edematous; no evidence of purulence or abscess Which empiric antibiotic should be recommended for this patient?

B. Cephalexin Unilateral lower extremity erythema, edema, and warmth are symptoms of cellulitis, a spreading infection of the dermis and subcutaneous fat that occurs when bacteria breach the skin barrier. Predisposing risk factors include skin trauma (eg, insect bite, abrasion), skin inflammation (eg, eczema), and comorbidities (eg, diabetes mellitus, obesity). Cellulitis can be purulent (eg, an abscess with or without drainage) or nonpurulent. Nonpurulent cellulitis in patients without signs of systemic toxicity (eg, fever, leukocytosis) can be treated with oral antibiotics that target streptococci (eg, Streptococcus pyogenes) and methicillin-susceptible Staphylococcus aureus (MSSA). Cephalexin or dicloxacillin is the preferred agent; clindamycin is an alternative in patients with a beta-lactam allergy. (Choice A) Amoxicillin (an aminopenicillin) has activity against streptococci, enterococci, and select gram-negative bacilli (eg, Escherichia coli, Klebsiella spp.), but it lacks the necessary activity against MSSA. (Choice C) Macrolides (eg, clarithromycin) are not recommended for nonpurulent cellulitis due to high rates of streptococcal resistance. (Choice D) Ciprofloxacin (a fluoroquinolone) has limited activity against gram-positive pathogens such as streptococci and MSSA. It is primarily used to empirically treat infections caused by gram-negative bacteria, including Pseudomonas aeruginosa. (Choice E) Metronidazole targets only anaerobic pathogens (eg, Bacteroides spp.). It may be included in empiric treatment regimens for polymicrobial skin and soft tissue infections (eg, diabetic foot infections), whereby broad-spectrum antibiotics are needed, but it has no role in the empiric management of cellulitis. Things to remember:Nonpurulent cellulitis (eg, no drainage or abscess) is commonly caused by streptococci and methicillin-susceptible Staphylococcus aureus. First-line outpatient treatment options include oral cephalexin or dicloxacillin. Patients with a beta-lactam allergy can receive clindamycin.

A 24-year-old male with no known health conditions comes to the community pharmacy to consult with the pharmacist about "bothersome" respiratory symptoms he has been experiencing over the past few days. The patient reports a mild ("scratchy") sore throat, a runny nose, and sneezing. His maximum temperature at home is 99.7°F (37.6°C). He does not have any shortness of breath, chest pain, itchy nose/eyes, loss of taste/smell, cough, or muscle aches. Which condition does this patient most likely have?

B. Common cold The common cold is an upper respiratory illness caused by one of several viruses (eg, rhinovirus, seasonal coronavirus). Symptoms include rhinorrhea, sneezing, mild sore throat, congestion, and cough. Fever is uncommon in adults but can occur in children. Diagnosis is based on these readily identifiable symptoms without the need for further testing. The common cold can be distinguished from other upper respiratory illnesses, which have their own complement of symptoms: Influenza: High fever, myalgia, fatigue, cough, and headache (Choice D) Streptococcal pharyngitis: Sudden-onset sore throat, swollen lymph nodes, patchy tonsillar exudates, lack of rhinorrhea and congestion (Choice E) The common cold is self-limiting with symptoms usually spanning 7-10 days. Over-the-counter medications can be used to target the specific symptoms and may include decongestants (eg, pseudoephedrine), antihistamines (eg, diphenhydramine), cough suppressants (eg, dextromethorphan), or expectorants (eg, guaifenesin). (Choices A and C) Acute bronchitis and community-acquired pneumonia (CAP) are lower respiratory tract infections. Acute bronchitis is characterized by a cough that lasts 1-3 weeks with or without sputum production; systemic symptoms (eg, fever, chills, malaise) and chest x-ray abnormalities are absent. CAP is characterized by fever, shortness of breath, chest pain, and purulent sputum; a chest x-ray showing infiltrates/consolidations is diagnostic. Things to remember:The common cold is a self-limiting (7-10 days) upper respiratory illness caused by one of several viruses (eg, rhinovirus). It can be differentiated from other respiratory illnesses based on its mild nature and lack of systemic symptoms (eg, fever, mylagia). Colds are treated with over-the-counter medications (eg, decongestants, antitussives) that target specific symptoms.

A 26-year-old male visits his primary care provider because of a rash on his upper left arm that began 3 days ago and appears to be expanding. The rash is mildly itchy but not painful. Other symptoms reported include fatigue and muscle aches. The patient has had no recent sick contacts. He returned from a hiking trip in Maine 10 days ago, during which he experienced a tick bite. Past Medical History: irritable bowel syndrome with diarrhea Allergies: NKDA Medications: loperamide 2 mg PO before meals, dicyclomine 10 mg PO QID PRN, amitriptyline 25 mg PO QHS Vital Signs: BP 116/70 mmHg, HR 76 bpm, RR 12 bpm, T 98.9°F (37.2°C), Ht 5' 11", Wt 75 kg Physical Exam: erythematous, circular rash on the upper left extremity, with clear center and a "bull's-eye" appearance; no cervical lymphadenopathy or neurological deficits

B. Doxycycline Lyme disease is a spirochetal infection caused by the transmission of Borrelia burgdorferi via a tick bite. An early clinical finding and hallmark feature of the disease is erythema migrans (EM), which typically appears 7-14 days after the tick bite. EM begins as a uniformly red rash that expands outward as it spreads through the dermis and has central clearing such that it resembles a bull's eye. Other symptoms of early-stage disease (within one month of the tick bite) mirror a viral syndrome (eg, fatigue, headache, myalgia). The first-line treatment for Lyme disease is oral doxycycline, amoxicillin, or cefuroxime. Treatment helps alleviate signs and symptoms and prevent progression to a later stage of the disease. Late-stage disease involves dissemination (eg, multiple EM lesions, lymphadenopathy), organ damage (eg, heart block, meningitis), and chronic arthritis or neuropathies. (Choices A, C, and D) Cephalexin (a first-generation cephalosporin), levofloxacin (a fluoroquinolone), and metronidazole (an antiprotozoal) do not have activity against Borrelia spp. and are not effective treatments for Lyme disease. Cephalexin covers staphylococci, streptococci, and select gram-negative pathogens (eg, Escherichia coli, Proteus); levofloxacin covers Streptococcus pneumoniae, gram-negative (eg, E. coli, Pseudomonas), and atypical organisms (eg, Mycoplasma); metronidazole covers only gram-negative anaerobes (eg, Bacteroides fragilis). (Choice E) Observation is not appropriate because it places the patient at risk for disseminated disease and long-term complications (eg, facial nerve palsy, arthritis). Things to remember:Lyme disease occurs when a tick bite transmits the spirochete Borrelia burgdorferi. A classic clinical finding is erythema migrans, which is a characteristic bull's-eye rash. Treatment with oral doxycycline, amoxicillin, or cefuroxime is essential to prevent disseminated disease and long-term complications (eg, chronic arthritis).

A patient with newly diagnosed latent tuberculosis comes to the pharmacy with a prescription for rifampin 600 mg daily for 4 months. Which of the following drug safety issues should the pharmacist mention when counseling about this medication? (Select ALL that apply)

B. Drug interactions C. Liver damage D. Orange-red body fluid discoloration Rifampin is an antibacterial drug that is well absorbed, has a wide tissue distribution, and can reach high intracellular concentrations. Although it is active against a variety of pathogens, it is most commonly used to treat mycobacterial (eg, Mycobacterium tuberculosis) infections and invasive Staphylococcus aureus infections (eg, endocarditis), often in combination with other antibiotics. Patients prescribed rifampin should be counseled regarding the following safety issues: Drug interactions: Rifampin induces multiple cytochrome P450 (CYP450) isoenzymes; most notably, it is a strong inducer of CYP3A4 and P-glycoprotein. Patients should speak with a healthcare provider when starting or stopping rifampin, or when a new medication is prescribed while taking rifampin (Choice B). Hepatotoxicity: Risk factors include preexisting liver disease, alcohol use disorder, or taking other hepatotoxic drugs (eg, isoniazid). Liver function tests should be monitored and patients should report any signs of severe hepatoxicity, including nausea, vomiting, dark urine, abdominal pain, fatigue, decreased appetite, or yellowing of the skin and eyes (Choice C). Orange-red discoloration of body fluids: This is a harmless adverse effect that can impact urine, saliva, sweat, and tears, and cause staining of contact lenses, bedsheets, and clothing (Choice D). Gastrointestinal upset (eg, nausea, vomiting) and hemolytic anemia (detected with a positive Coombs test). (Choices A and E) Acute gouty attacks are a risk with pyrazinamide, and vision damage can occur with ethambutol; both are used in active tuberculosis treatment regimens. Things to remember:Patients taking rifampin should be counseled about the many safety issues involved, including the risk for drug interactions, hepatotoxicity, orange-red body fluid discoloration (which is harmless), and hemolytic anemia.

A patient comes to the pharmacy reporting new-onset blurred vision that began within the past week. According to his pharmacy profile, the following medications were dispensed recently: Last fillRxMedicationQuantityRefills 11/15RefillDolutegravir 50 mg 1 tab PO BID605 11/15RefillEmtricitabine 200 mg/tenofovir disoproxil fumarate 300 mg 1 tab PO daily305 11/15RefillSulfamethoxazole/trimethoprim 400 mg/80 mg 1 tab PO daily 30 5 11/05 New Pyrazinamide 500 mg 4 tab PO daily 120 1 11/05 New Ethambutol 400 mg 4 tab PO daily 120 1 11/05NewRifampin 600 mg 2 cap PO daily 60 5 11/05NewIsoniazid 300 mg 1 tab PO daily 30 5 10/21NewDoxycycline monohydrate 100 mg 1 tab PO BID 14 0 Which of the following medications is most likely responsible for the patient's symptoms?

B. Ethambutol This patient is taking ethambutol, in combination with isoniazid, rifampin, and pyrazinamide (commonly abbreviated as RIPE) to treat active tuberculosis. The package labeling for ethambutol contains a warning for visual damage, specifically optic neuritis, which can manifest as decreased acuity (eg, blurred vision), vision loss, color blindness, or another visual defect. Damage can occur in one eye (unilateral) or both eyes (bilateral) and is usually reversible after drug discontinuation (may take up to 1 year for vision to return to baseline). Patients prescribed ethambutol require visual examinations (eg, visual acuity, color discrimination) at baseline (before starting the medication) and then periodically (eg, monthly) while therapy continues. If patients experience vision changes, they should notify their healthcare provider immediately. (Choice A) The timing of symptom onset and date that doxycycline was dispensed make it unlikely that doxycycline is the cause of the patient's blurred vision because the seven-day course would have been completed before symptoms began. (Choice C) Pyrazinamide can increase uric acid levels and is contraindicated in acute gout. Like all RIPE therapies, it can also cause hepatoxicity. (Choice D) Rifampin is known to cause hepatotoxicity (elevated liver function tests), orange-red discoloration of body fluids (eg, tears, sweat, urine), a flu-like syndrome, and hematologic effects (eg, hemolytic anemia). (Choice E) Sulfamethoxazole/trimethoprim can cause photosensitivity, hyperkalemia, hemolytic anemia, and crystalluria (prevented by taking with 8 oz of water). Things to remember:Ethambutol carries a labeled warning for optic neuritis, a type of vision damage that presents as blurred vision, vision loss, color blindness, or another visual defect. Monitoring is required with baseline and periodic visual examinations until the medication is discontinued.

WP is a 70-year-old male who comes to the emergency department with a 3-day history of abdominal tenderness, decreased appetite, and 6 to 7 watery stools per day. Laboratory and diagnostic studies show a white blood cell count of 17,000 cells/mm3, serum creatinine of 1.8 mg/dL (baseline: 0.8 mg/dL), and stool test positive for Clostridioides difficile. Six weeks ago, WP experienced a first episode of C. difficile infection, which was treated with a 10-day course of oral vancomycin 125 mg four times daily. Which of the following are recommended treatment options for this patient? (Select ALL that apply)

B. Fidaxomicin 200mg PO BID E. Vancomycin PO as a tapered and pulsed regimen Recurrent Clostridioides difficile infection (CDI) is a reappearance of symptoms within 2-8 weeks of a prior CDI episode. After an initial episode, the rate of recurrence is approximately 20-25% and can be higher in patients with risk factors (age ≥ 65 years, immunocompromised status, severe CDI presentation). The preferred treatment options for a second CDI episode (first recurrence) include oral fidaxomicin or an oral vancomycin tapered and pulsed regimen. Both treatments have lower rates of recurrence compared to a standard oral vancomycin regimen without a taper, which should be considered only when the initial CDI episode was treated with metronidazole. In patients with a prior CDI episode in the past 6 months, bezlotoxumab may be administered in addition to antibiotics to neutralize C. difficile toxin B and decrease the risk for further recurrences. (Choice A) Fecal microbiota transplantation can be considered after multiple CDI episodes (second or subsequent recurrences) in patients who previously received recommended CDI antibiotic regimens. (Choice C) Oral metronidazole is not recommended for the treatment of severe CDI (any occurrence) because it has demonstrated inferiority compared to other available treatments. It can be used for an initial episode of nonsevere CDI if oral fidaxomicin or oral vancomycin is unavailable. (Choice D) High-dose oral vancomycin does not improve clinical outcomes (cure rates, recurrence rates) compared to standard doses in patients with severe CDI. High-dose vancomycin, typically in combination with intravenous metronidazole, is recommended only in patients with fulminant CDI. Things to remember:Oral fidaxomicin or a standard oral vancomycin regimen followed by a prolonged taper is the preferred treatment for a second episode (first recurrence) of Clostridioides difficile infection.

A 23-year-old sexually active male visits his primary care physician concerned about multiple soft, nonpainful, mildly pruritic, skin-colored papules that have developed on the shaft of his penis. He is diagnosed with genital warts. What treatment is recommended?

B. Imiquimod cream Anogenital warts are a sexually transmitted disease caused by human papillomavirus (HPV) strains that are low risk for causing cancer (nononcogenic). If left untreated, the warts generally resolve spontaneously within one year. Vaccination significantly reduces the incidence of anogenital warts and cancers caused by HPV (eg, cervical cancer). Imiquimod cream is an immune activator that stimulates production of interferon and other cytokines that help resolve the appearance of the warts and reduce HPV viral loads. The cream should be applied to clean, dry, warty tissue until it is completely absorbed and should be washed off with soap and water 6-10 hours after application. Local reactions to imiquimod include erythema, burning, scaling, crusted skin, ulcers, and vesicles. Patients should abstain from all sexual contact while the cream is on the skin because it can weaken condoms and diaphragms and irritate the anogenital mucosa. (Choices A and C) Oral clindamycin (which covers gram-positive aerobes and anaerobes) and metronidazole gel (which has anaerobic antibacterial activity only) are treatment options for bacterial vaginosis in females. (Choice D) Mupirocin ointment is used to treat impetigo (a superficial skin infection) and can also be applied to the nares to decolonize patients with methicillin-resistant Staphylococcus aureus (MRSA). (Choice E) Tinidazole is a treatment alternative for bacterial vaginosis and trichomoniasis and can be used as an alternative to metronidazole in Helicobacter pylori treatment regimens. Things to remember:Human papillomavirus is a sexually transmitted disease that causes anogenital warts. Imiquimod cream is an immune activator that can help resolve the appearance of the warts.

TL is a 19-year-old female who comes to the university health clinic with increased vaginal discharge for the last 2 days. The discharge is thin, gray, and has a fishy odor. A vaginal swab shows a pH of 5.2 and findings consistent with bacterial vaginosis. Which treatment is recommended?

B. Intravaginal metronidazole Vaginal discharge is a common symptom reported by women, and the specific description (color, consistency, odor) can indicate the presence of a vaginal infection. Bacterial vaginosis (BV) occurs when normal vaginal flora is replaced with anaerobes (eg, Gardnerella vaginalis), resulting in a thin, gray (off-white) discharge with a fishy odor and elevated pH (above 4.5). Other features of inflammation (eg, vaginal redness, pruritus) are typically absent; this is the reason it is called bacterial vaginosis and not vaginitis. The recommended treatment for BV includes metronidazole (intravaginal or oral) or intravaginal clindamycin. Treatment with topical (eg, intravaginal) agents is specific to BV and differs from the treatment of trichomoniasis, which requires systemic drugs (eg, oral metronidazole). (Choice A) Ceftriaxone (a third-generation cephalosporin) has gram-positive (eg, streptococci) and gram-negative (eg, Neisseria gonorrhoeae) activity but does not cover anaerobes and is not an effective treatment for BV. (Choice C) Intravaginal miconazole (an OTC antifungal) is used to treat vaginal candidiasis, which typically presents with a thick, white, odorless, cottage cheese-like discharge and pruritus. Because self-diagnosis of vaginal candidiasis is often inaccurate, self-treatment is discouraged as it can delay effective therapy for BV or other vaginal infections. (Choice D) Azithromycin (a macrolide) does not have adequate anaerobic activity to treat BV. It is the drug of choice when treating chlamydial infections in pregnant women. (Choice E) Imiquimod is an immune activator, not an antimicrobial. It is a topical treatment option for external anogenital warts caused by nononcogenic strains of human papillomavirus. Things to remember:Symptoms of bacterial vaginosis include a thin, gray, vaginal discharge with a fishy odor and pH > 4.5. It can be treated topically with intravaginal metronidazole or clindamycin, or systemically with oral metronidazole.

SD is a 48-year-old female preparing to start Humira for rheumatoid arthritis. Required pre-treatment screening for latent tuberculosis (TB) reveals a positive interferon-gamma release assay. A chest x-ray is negative, and the patient reports no signs or symptoms of active pulmonary TB. Which treatment should this patient initiate before starting Humira?

B. Isoniazid + rifampin daily for 3 months Tumor necrosis factor-alpha (TNF-alpha) is an essential component of the immune response that recruits cells (eg, macrophages) to sequester and contain Mycobacterium tuberculosis during the latent phase of the disease. TNF-alpha inhibitors [eg, Humira (adalimumab), Remicade (infliximab)], used to treat inflammatory conditions such as rheumatoid arthritis, can cause reactivation of latent tuberculosis (TB). For this reason, all TNF-alpha inhibitors have a boxed warning to screen for latent TB before initiation, preferably with an interferon-gamma release assay blood test. Patients who test positive and have no evidence of active TB (eg, no signs/symptoms, negative chest x-ray) should start one of the following preferred latent TB treatment regimens before TNF-alpha inhibitor therapy is initiated: Isoniazid and rifapentine weekly for 12 weeks Isoniazid and rifampin daily for 3 months Rifampin daily for 4 months (Choice A) Isoniazid monotherapy is an alternative latent TB regimen, but the recommended treatment duration is 6 or 9 months, and it has a higher risk of hepatotoxicity and medication nonadherence compared to the shorter (3-4 months) rifamycin-based regimens. (Choices C and D) The recommended latent TB treatment duration is 12 weeks for the combination of isoniazid and rifapentine and 4 months for rifampin monotherapy. (Choice E) The combination of rifampin and pyrazinamide should not be used to treat latent TB because of the risk of severe hepatotoxicity. Things to remember:Screening for latent tuberculosis is required before starting a tumor necrosis factor-alpha (TNF-alpha) inhibitor (eg, adalimumab, infliximab). Patients who test positive should be initiated on latent TB treatment (eg, isoniazid and rifampin daily for 3 months) before TNF-alpha inhibitor treatment is started.

SL is a 25-year-old pregnant female in her second trimester who has had close contact with a family member diagnosed with active pulmonary tuberculosis. A tuberculin skin test is administered, and the test interpretation 48 hours later is positive based on the measured area of induration. The patient has no symptoms of active tuberculosis, and a chest x-ray is negative. Which treatment is appropriate for SL?

B. Isoniazid and rifampin daily for 3 months Tuberculosis (TB), caused by Mycobacterium tuberculosis, has two stages: latent and active disease. With latent TB, the infection is contained by host defenses and the patient is asymptomatic, has a normal chest x-ray, and is noninfectious. Testing for latent TB should be performed in patients with an increased risk of new TB infection (eg, contact with an untreated respiratory TB case) or reactivation (eg, immunocompromising conditions, tumor necrosis factor-alpha inhibitor treatment). Latent TB treatment options for pregnant patients include: Isoniazid (INH) plus rifampin daily for 3 months Rifampin daily for 4 months INH daily for 6 or 9 months A fourth option, INH plus rifapentine once weekly for 12 weeks, is not recommended in pregnant individuals due to unknown effects on the fetus. Shorter rifamycin-based regimens are preferred to 6 or 9 months of INH because of improved adherence and reduced hepatotoxicity. Pyridoxine (vitamin B6) should be added to INH-containing regimens to reduce the risk of peripheral neuropathy. (Choice A) Isoniazid monotherapy is an option for latent TB but requires a longer treatment duration (6 or 9 months). (Choice C) Nine months of INH plus rifampin does not convey additional clinical benefit and increases the risk for toxicity compared to the recommended duration of 3 months. (Choices D and E) Treatment with rifampin, INH, pyrazinamide, and ethambutol (RIPE) is an option for active (not latent) TB. Active TB requires more drugs because of the risk of developing resistance while on therapy. Things to remember:Treatment options for latent tuberculosis in pregnant patients include isoniazid plus rifampin daily for 3 months, rifampin daily for 4 months, or isoniazid daily for 6 or 9 months.

A 44-year-old male is evaluated after he was exposed to an individual with active pulmonary tuberculosis. He is asymptomatic and has no history of bacille Calmette-Guérin vaccination. Which of the following diagnostic test results would be expected in a patient with latent tuberculosis? (Select ALL that apply)

B. Normal chest x-ray D. Positive interferon-gamma release assay blood test E. Positive intradermal tuberculin skin test reaction An initial Mycobacterium tuberculosis infection that is suppressed by the host is called latent tuberculosis (TB); the patient is asymptomatic and noncontagious. Because the disease can become active and infectious at any time, screening for latent TB in at-risk patients (eg, exposed to active pulmonary TB, immunocompromised, HIV positive) is essential for identifying the need for treatment, which significantly reduces the risk of conversion to active disease and subsequent transmission to others. A latent TB diagnosis should meet both of the following criteria: A positive interferon-gamma release assay (IGRA) blood test or tuberculin skin test (TST) (Choices D and E). Active disease is excluded (the patient is asymptomatic and has a negative chest x-ray) (Choice B). Because a TST requires follow-up in 48-72 hours to measure the area of skin induration (ie, test response), and a false-positive result can occur with a history of bacille Calmette-Guérin vaccination, an IGRA blood test is preferred for most patients. (Choices A and C) Positive chest x-ray findings (eg, infiltrates, cavitation) and a positive acid-fast bacilli sputum smear are signs of active pulmonary TB. The presence of clinical symptoms (eg, cough, fever, night sweats, weight loss), risk factors (eg, recent exposure to a person with untreated active disease), a positive TST or IGRA test result, and a positive sputum culture can aid in confirming the diagnosis. Things to remember:Patients with risk factors (eg, recent exposure to an individual with active TB) should be screened for latent tuberculosis, which is diagnosed if the patient is asymptomatic, has a negative chest x-ray, and a positive tuberculin skin test or interferon-gamma release assay blood test.

A 5-year-old girl is evaluated for a new-onset rash. Four days ago, a few small "bumps" appeared around the corners of the patient's mouth, which soon developed into "blisters" that burst to form thick honey-colored crusts. The rash is mildly itchy and is spreading quickly, with multiple lesions now evident around the mouth, cheeks, and nose. Vital signs are normal. Which of the following is the most appropriate treatment for this condition?

B. Oral cephalexin Impetigo is a contagious superficial skin infection most commonly caused by Staphylococcus aureus and group A Streptococcus (S. pyogenes). It commonly affects young children and usually begins with papules and pustules on the face or extremities. Over approximately one week, the pustules rupture and harden into characteristic golden-yellow (honey-colored)crusts. Impetigo may be mildly itchy and/or painful. Lesions tend to stay localized but can spread quickly through touch (frequent hand hygiene is recommended). Antibiotics are indicated to reduce transmission and recovery time. Topical antibiotics (eg, mupirocin) are adequate for limited, localized infections. Oral antibiotics (eg, cephalexin, dicloxacillin) are warranted when there are numerous and/or extensive lesions, which makes topical therapy impractical. (Choice A) Amoxicillin does not have adequate activity against S. aureus and therefore is not recommended for impetigo. (Choice C) Topical docosanol is indicated for the treatment of cold sores (caused by the herpes simplex virus). (Choice D) Topical corticosteroids (eg, hydrocortisone) are indicated to treat eczema (atopic dermatitis), which is commonly caused by environmental irritants (eg, allergens) and presents with an erythematous, dry, scaly rash that is extremely itchy. (Choice E) Warm compresses alone are not effective at treating impetigo. They may be used to aid in the removal of crusts, but because the disease is highly contagious, any cloth used for this purpose should be washed with detergent and hot water and dried with high heat. Things to remember:Impetigo is a contagious, superficial skin infection of the face and/or extremities that begins with papules/pustules and progresses to a characteristic overlying honey-colored crust. Oral antibiotics (eg, cephalexin) are indicated for more extensive lesions because topical therapy is impractical.

A female patient comes to the pharmacy counter to pick up a prescription for nitrofurantoin. She is also purchasing a box of Azo Urinary Pain Relief. Which of the following are appropriate counseling points for the over-the-counter medication being purchased? (Select ALL that apply)

B. Take with food to minimize stomach upset C. The maximum duration of use is two days D. This medication can cause urine to turn a red-orange color E. This medication helps symptoms but does not treat infection Phenazopyridine contains an azo dye that has anesthetic and analgesic effects on urinary tract mucosa, although its exact mechanism of action is unknown. It is often recommended to relieve symptoms of urinary pain, burning, urgency, and frequency associated with acute cystitis (a lower urinary tract infection) while awaiting the effects of the prescribed antibiotic, which can take up to 48 hours. Phenazopyridine is available over-the-counter (eg, Azo Urinary Pain Relief) or with a prescription (eg, Pyridium), and should be taken three times daily with a full glass of water. Key counseling points include: Take with food to decrease adverse gastrointestinal effects (eg, stomach cramps) (Choice B). The maximum duration of use is 2 days (Choice C). Red-orange discoloration of urine can occur; it may stain clothing or contact lenses (Choice D). It relieves symptoms of a urinary tract infection, but does not treat the infection (Choice E). (Choice A) Phenazopyridine does not have any clinically relevant food or drug interactions and can be taken concurrently with dairy products. Examples of medications that should be separated from dairy (ie, calcium-containing) products to avoid binding interactions and decreased absorption include tetracyclines, fluoroquinolones, oral bisphosphonates, and levothyroxine. Things to remember:Phenazopyridine (eg, Azo Urinary Pain Relief) is a urinary analgesic used to relieve lower urinary tract infection symptoms (eg, urinary pain, burning) while awaiting the effects of an antibiotic (which treats the infection). Patients should be counseled to take with food to decrease stomach upset, use no longer than 2 days, and expect red-orange discoloration of urine.

A 52-year-old male develops chills, a fever of 101°F (38.3°C), and an elevated white blood cell count of 14,200 cells/mm3. Blood cultures are positive for Enterococcus faecalis, and an echocardiogram shows a mitral valve vegetation. The patient is receiving high-dose penicillin G and gentamicin. Which of the following best describes why this combination of antibiotics is being used to treat the infection?

B. To enhance intracellular drug penetration Synergy occurs when the combined effects of two antibiotics (eg, an aminoglycoside and a beta-lactam) are greater than the sum of the individual drug effects. This strategy is often employed for infections that are difficult to eradicate, such as prosthetic valve endocarditis or endocarditis caused by Enterococcus faecalis. Penicillin G works by disrupting the stability of the bacterial cell wall, which allows aminoglycosides (eg, gentamicin) to more readily enter the cell and inhibit protein synthesis. The result is an enhanced bactericidal effect. (Choice A) Both penicillin G and gentamicin target E. faecalis and thus do not work together to broaden the activity. Aminoglycosides alone cannot pass through the thicker cell wall of gram-positive organisms, which is why penicillin G is necessary. An example of broadening the spectrum of activity would be adding an antibiotic with gram-negative activity (eg, aztreonam) to one with gram-positive activity (eg, vancomycin). (Choice C) Currently, there are no medications that work to inhibit the efflux of antibiotics out of the cell. (Choice D) Beta-lactamase inhibitors (eg, clavulanate) prevent enzymatic degradation of beta-lactams (eg, amoxicillin). The inhibitor serves as a decoy for the bacterial enzyme, allowing the beta-lactam to escape degradation and exert its mechanism. (Choice E) Probenecid increases plasma levels of penicillin and other beta-lactams by competitively inhibiting their secretion via the renal tubules, thereby reducing urinary excretion of the antibiotic. Things to remember:Synergy results when a combination of antibiotics improves bacterial killing beyond the additive effects of the individual drugs. An example is adding a cell wall-active antibiotic (eg, penicillin G) to an aminoglycoside (eg, gentamicin), which improves intracellular aminoglycoside penetration when treating gram-positive infections.

LW is a 48-year-old male who comes to the emergency department with fever, abdominal pain, and mild confusion for the past 3 days. He is accompanied by his wife, who reports an increase in the patient's abdominal girth over the past 3-4 weeks but no vomiting of blood, diarrhea, or bloody stools. Past Medical History: alcohol-associated cirrhosis, ascites, portal hypertension, hepatic encephalopathy Social History: occasional alcohol use (4-6 beers on weekends), does not use tobacco Allergies: NKDA Home Medications: spironolactone 200 mg PO daily, furosemide 80 mg PO daily, lactulose 30 mL PO TID, nadolol 40 mg PO daily Vital Signs: BP 110/60 mmHg, HR 68 bpm, RR 18 bpm, T 100.8°F (38.2°C), Ht 5′ 9″, Wt 96 kg (increase of 7 kg) Laboratory Tests:Sodium 133 mEq/LSerum creatinine 1.4 mg/dLBlood urea nitrogen 28 mg/dLAlbumin 2.6 g/dLTotal bilirubin 3.2 mg/dLAST 67 units/LALT 58 units/LAlkaline phosphatase 120 units/LINR 1.4White blood cells 14,200 cells/mm3Hemoglobin 9.2 g/dLPlatelets 120,000 cells/mm3 Diagnostic Tests:Diagnostic paracentesis, ascitic fluid analysis: total protein 0.9 g/dL, albumin 1.6 g/dL, polymorphonuclear leukocyte count 450 cells/mm3Ascitic fluid culture: pending Additional Information: moderate ascites, start empiric antibiotics for spontaneous bacterial peritonitis. Which antibiotic regimen is preferred to manage this patient's condition?

C. Ceftriaxone for 5 days Spontaneous bacterial peritonitis (SBP) is a common complication of cirrhosis and most often occurs in patients with ascites (ie, fluid accumulation in the peritoneal space). Infection arises when gastrointestinal bacteria (eg, Escherichia coli, Klebsiella pneumoniae) translocate into ascitic fluid via the lymphatic system. Patients can exhibit symptoms (eg, fever, abdominal pain, altered mental status) or be asymptomatic. A paracentesis should be performed promptly to collect a sample of ascitic fluid for analysis, including a cell count and differential. A polymorphonuclear leukocyte count ≥ 250 cells/mm3 suggests the presence of an infection, and empiric antibiotics should be started until ascitic fluid culture results are available. Recommended antibiotics include ceftriaxone or cefotaxime for 5-7 days. A broader-spectrum agent (eg, a carbapenem) is appropriate for critically ill patients or those at risk for resistance. (Choice A) Although the combination of ampicillin and tobramycin targets the common causative pathogens, this regimen is less effective than a third-generation cephalosporin and has a higher rate of nephrotoxicity. Nephrotoxic drugs (eg, tobramycin) should be avoided because patients with cirrhosis, ascites, and SBP are predisposed to develop renal failure. (Choice B) Cefepime (a fourth-generation cephalosporin) is typically reserved for infections caused by nosocomial pathogens (eg, Pseudomonas aeruginosa). Because anaerobes are not a common cause of SBP, metronidazole is not needed. (Choices D and E) Clindamycin has activity against gram-positive organisms (eg, staphylococci, streptococci) and anaerobes, while metronidazole covers only anaerobic pathogens. Neither antibiotic targets the most common pathogens that cause SBP. Things to remember:Spontaneous bacterial peritonitis, suspected when the ascitic fluid polymorphonuclear leukocyte count is ≥ 250 cells/mm3, is treated empirically with ceftriaxone or cefotaxime.

An 18-month-old female is brought to the office by her mother, who reports that her child has been tugging on both ears, eating less, and crying more than usual. In the office, the child has a temperature of 101.5°F (38.6°C) and examination reveals bulging bilateral tympanic membranes and middle ear effusions. The patient is allergic to penicillin, which caused a mild nonpruritic rash on her torso and arms three days after initiation. Which antibiotic is preferred to empirically treat the acute infection?

C. Cefuroxime Bulging tympanic membranes and middle ear effusions are hallmark otoscopic findings of acute otitis media (AOM). Common AOM symptoms in young children include ear pain and tugging, fever, decreased appetite, and irritability. Patients who meet criteria for antibiotics (eg, age < 2 years with bilateral infection) should receive an agent with activity against the common pathogens (eg, Streptococcus pneumoniae, Haemophilus influenzae, Moraxella catarrhalis). The first-line treatment for AOM is amoxicillin, or amoxicillin-clavulanate, 90 mg/kg/day. In patients with a mild penicillin allergy (eg, a delayed-onset maculopapular rash), a second- or third-generation cephalosporin (eg, cefuroxime) has a low risk of cross-reactivity and is the recommended alternative. (Choice A) Aminopenicillins (eg, amoxicillin/clavulanate) are frequently prescribed and cause delayed maculopapular rash in 5-10% of children. Patients with this type of mild hypersensitivity reaction to a penicillin can be treated with a nonpenicillin beta-lactam (eg, a cephalosporin). (Choices B and E) Because of a higher prevalence of macrolide- (eg, azithromycin) and clindamycin-resistant S. pneumoniae and limited activity of either antibiotic against H. influenzae, use of azithromycin or clindamycin should be reserved for children with a severe penicillin allergy (eg, anaphylaxis). (Choice D) Ciprofloxacin has no reliable activity against S. pneumoniae and should not be used to treat AOM. Topical ciprofloxacin formulations (eg, ear drops) are appropriate for acute otitis externa, where the common causative pathogens are Staphylococcus aureus and Pseudomonas aeruginosa. Things to remember:The recommended treatment for acute otitis media in children with a mild, delayed-onset hypersensitivity reaction (eg, a maculopapular rash) to penicillin is a second- or third-generation cephalosporin (eg, cefdinir, cefpodoxime, cefuroxime).

A 36-year-old male was newly diagnosed with active pulmonary tuberculosis 2 months ago. He has completed an initial 8 weeks of treatment with isoniazid, rifampin, pyrazinamide, and ethambutol and is now being evaluated at a follow-up visit. His baseline sputum culture results at the time of diagnosis showed no evidence of drug resistance. The patient has tolerated the current 4-drug regimen, and his baseline symptoms of fever, cough, and night sweats have resolved. Which of the following represents an appropriate treatment plan for this patient?

C. Continue isoniazid and rifampin for 4 months Adults with active tuberculosis (TB) are initially treated with a 2-month (8-week), intensive phase regimen consisting of four drugs: rifampin, isoniazid, pyrazinamide, and ethambutol (known as RIPE therapy). Symptoms (eg, fever, cough, night sweats) typically improve within 2-3 weeks and can indicate drug susceptibility, though this should be confirmed with final culture results. Because the initial sputum culture results showed no evidence of drug resistance, this patient should receive continuation phase treatment with isoniazid and rifampin for at least 4 months. (Choice A) Studies evaluating the appropriate duration of treatment for TB have demonstrated that a 2-drug regimen with isoniazid and rifampin is sufficient after the initial 2 months, and that there are no benefits—but possible adverse effects (eg, hepatotoxicity)—in continuing all four RIPE drugs. (Choice B) Ethambutol and pyrazinamide are used in the initial, 2-month intensive treatment phase as part of RIPE therapy. Once susceptibility to isoniazid and rifampin is confirmed, they have no role and can be discontinued. (Choice D) Isoniazid monotherapy does not constitute an adequate regimen and could lead to drug resistance and/or treatment failure. (Choice E) Active TB is both damaging to the host (patient) and highly contagious to others. A total treatment duration of 2 months (instead of 6 months) is not sufficient to eradicate Mycobacterium tuberculosis and could result in treatment failure, disease progression (eg, worsening lung function), and a higher risk of community transmission. Things to remember:Active tuberculosis requires treatment for at least 6 months: 2 months of intensive phase treatment with four drugs (rifampin, isoniazid, pyrazinamide, and ethambutol) followed by at least 4 months of continuation phase treatment with isoniazid and rifampin.

AW is a 68-year-old female who visits her primary care provider due to swelling and redness on her left calf. The patient initially noticed an insect bite on her calf a few days ago. The area of redness and swelling has grown and there is now a painful "lump" present. Past Medical History: hypothyroidism, hypertension, osteoporosis, throat cancer (5 years ago, in remission) Allergies: sulfa (hives) Medications:Losartan 50 mg PO dailyLevothyroxine 75 mcg PO dailyProlia 60 mg SC every 6 monthsCaltrate 600+D3 one soft chew daily Vital Signs: BP 138/76 mmHg, HR 86 bpm, RR 12 bpm, T 100.5°F (38.1°C), Ht 5′ 1″, Wt 52 kg Physical Exam: 2.5-cm area of erythema on the left posterior lower extremity, with underlying fluctuance and induration of the surrounding skin; tender to touch but no drainage noted Additional Information: Abscess incision and drainage performed with 5 mL of purulent fluid collected. Culture and susceptibility pending. Start empiric antibiotics.

C. Doxycycline The clinical findings of unilateral lower extremity swelling, erythema, pain, and fluctuance are consistent with a skin abscess, a type of purulent cellulitis that is predominantly caused by methicillin-susceptible Staphylococcus aureus (MSSA) or methicillin-resistant S. aureus (MRSA). Predisposing risk factors include skin trauma (eg, insect bite, skin abrasion), injection drug use, and comorbid conditions (eg, diabetes mellitus, obesity, venous insufficiency). Purulent cellulitis is managed with source control (eg, skin incision and drainage of infected fluid/pus) followed by empiric antibiotics with activity against MSSA and MRSA; outpatient treatment with doxycycline or sulfamethoxazole/trimethoprim is appropriate in patients without signs of systemic toxicity (eg, fever, chills, tachycardia). (Choices A and B) Dicloxacillin (an antistaphylococcal penicillin) and clarithromycin (a macrolide) lack activity against MRSA, which should be targeted when empirically treating purulent cellulitis. Dicloxacillin is an appropriate treatment for nonpurulent cellulitis, which is more likely to be caused by streptococci and MSSA; clarithromycin should not be used to empirically treat cellulitis due to high rates of streptococcal resistance. (Choice D) Penicillin V potassium has a limited spectrum of activity that includes streptococci, enterococci, and gram-positive anaerobes (eg, Peptostreptococcus). It has no activity against MSSA or MRSA. (Choice E) Although sulfamethoxazole/trimethoprim is a treatment of choice for purulent cellulitis, it should be avoided in patients with a documented sulfa allergy. Things to remember:Purulent cellulitis (eg, skin abscess and/or draining fluid/pus) is most commonly caused by methicillin-susceptible or methicillin-resistant Staphylococcus aureus. The preferred oral antibiotic for outpatient treatment of mild infection is doxycycline or sulfamethoxazole/trimethoprim (if no sulfa allergy is documented).

KH, a 24-year-old male who is sexually active with multiple men, comes to the office for a routine annual physical examination. His records indicate he has received all necessary vaccinations, including the human papillomavirus vaccine series. Routine screening for HIV, hepatitis C, and syphilis are negative. Urethral and rectal swabs sent for nucleic acid amplification testing are positive for Chlamydia trachomatis and negative for Neisseria gonorrhoeae. Which treatment is recommended for this patient?

C. Doxycycline 100mg by mouth twice daily for 7 days Chlamydial infections are often asymptomatic in both men and women. Screening is therefore recommended for sexually active women age ≤ 25, men who have sex with men, and any patient with risk factors (eg, multiple sex partners). Nucleic acid amplification testing of genital, rectal, and/or oropharyngeal swabs for Chlamydia trachomatis is a preferred diagnostic method. Because C. trachomatis replicates intracellularly, antibiotics that achieve high intracellular concentrations (eg, tetracyclines, macrolides) are more effective than those that concentrate in the intravascular space (eg, beta-lactams). The first-line treatment for uncomplicated chlamydial infections in nonpregnant adults is doxycycline 100 mg twice daily for 7 days. Doxycycline has a higher rate of microbiological cure than a single dose of azithromycin 1 gram, particularly for rectal infections. Azithromycin should only be used in pregnant women (because doxycycline is contraindicated) or when there is a concern for nonadherence with 7 days of therapy. (Choice A) Cephalosporins (including ceftriaxone) have no activity against C. trachomatis and are not indicated to treat chlamydial infections. Ceftriaxone is the first-line treatment for gonococcal infections. (Choices B and E) Clindamycin has activity against gram-positive organisms (eg, streptococci, staphylococci) and anaerobes, while metronidazole has activity against only anaerobic pathogens. Both are ineffective in treating infections caused by gram-negative bacteria such as C. trachomatis. Clindamycin and metronidazole are treatment options for sexually transmitted infections caused by anaerobic pathogens (eg, bacterial vaginosis). (Choice D) Erythromycin has some activity against C. trachomatis but is not recommended to treat confirmed infections due to a lower cure rate and a higher risk of gastrointestinal adverse effects that lead to nonadherence. Things to remember:The treatment of choice for chlamydial infections in men and nonpregnant women is doxycycline 100 mg twice daily for 7 days. Azithromycin 1 gram as a single dose is the preferred treatment in pregnant women.

CM is a 29-year-old sexually active male who comes to the emergency department after finding a painless, 2-cm ulcer on his penis. He does not have a fever or urethral discharge but discloses that he currently engages in unprotected sex. The patient has an allergy to amoxicillin (throat swelling).

C. Doxycycline PO Syphilis is a sexually transmitted infection (STI) caused by the spirochete Treponema pallidum. It may present at different stages (eg, primary, secondary, latent), which warrants different treatments. This patient has evidence of primary syphilis due to a history of unprotected sex, the presence of a chancre, and a positive nontreponemal (rapid plasma reagin) and treponemal assay. Primary syphilis is treated with a single dose of intramuscular penicillin G benzathine. An alternative regimen for nonpregnant patients with a penicillin allergy is oral doxycycline. Pregnant and nonadherent patients with a penicillin allergy should not receive doxycycline; they must undergo desensitization with subsequent administration of penicillin. (Choice A) Azithromycin (a macrolide) has activity against atypical pathogens (eg, Legionella, Chlamydia, Mycoplasma) but does not have reliable activity against T. pallidum due to increasing resistance. Azithromycin is used to treat chlamydia in special populations (eg, pregnancy). (Choice B) Ceftriaxone (a third-generation cephalosporin) has activity against T. pallidum and can be considered to treat primary syphilis (unlike late latent syphilis). However, patients with a penicillin allergy may be at risk for cross-reactivity and the optimal dose and duration for the treatment of primary syphilis is not well defined. (Choice D) Intramuscular penicillin G benzathine (Bicillin-LA) is the preferred treatment for primary syphilis; however, it should be avoided in this penicillin-allergic patient. (Choice E) Trimethoprim-sulfamethoxazole (a sulfonamide) does not treat syphilis because it has no activity against T. pallidum. It is commonly used to treat urinary tract infections and opportunistic infections (eg, Pneumocystis pneumonia). Things to remember:Intramuscular penicillin G benzathine (Bicillin L-A) is the first-line treatment for primary syphilis. In nonpregnant patients with a severe penicillin allergy, doxycycline is the preferred alternative

BT is a 28-year-old female with a 2-day history of increased urinary frequency and burning with urination. She has suprapubic tenderness but no flank pain. Her vital signs are normal. A urinalysis is positive for white blood cells, leukocyte esterase, and nitrites, and a urine culture is pending. BT has a history of Escherichia coli urinary tract infections, the last of which was treated 2 months ago with sulfamethoxazole/trimethoprim. She has no known drug allergies. Which antibiotic is the best choice to empirically treat the infection while awaiting culture results?

C. Fosfomycin Painful, frequent urination and suprapubic tenderness are symptoms of acute cystitis, a lower urinary tract infection. White blood cells, leukocyte esterase, and nitrites on a urinalysis support the diagnosis. Additional testing with a urine culture is recommended for recurrent cystitis (eg, ≥ 2 infections in 6 months) or if symptoms persist with initial treatment. Empiric treatment of acute cystitis targets the most likely pathogen, Escherichia coli, and other possible organisms (eg, Proteus spp., Klebsiella spp.). First-line options include sulfamethoxazole/trimethoprim, nitrofurantoin, or fosfomycin. Antibiotics that have been used within the past 3 months should be avoided empirically because resistance may have developed. Since this patient received sulfamethoxazole/trimethoprim 2 months ago, the preferred treatments are nitrofurantoin or fosfomycin. (Choice A) Aminopenicillins (eg, amoxicillin) are not recommended for empiric treatment of acute cystitis due to high rates of gram-negative bacterial resistance. (Choice B) Bactrim DS (sulfamethoxazole/trimethoprim double-strength) has a lower threshold for resistance compared to other first-line acute cystitis treatments and should not be selected if it has been used within the past 3 months. (Choice D) Metronidazole has activity against gram-negative anaerobes but lacks the necessary activity against gram-negative aerobes. (Choice E) Zithromax (azithromycin) has activity against gram-negative pathogens (eg, Haemophilus influenzae, Moraxella catarrhalis) that are more likely to cause respiratory (not urinary tract) infections. In addition, Zithromax is primarily excreted in the bile and feces and does not reach adequate concentrations in the urine to treat an infection. Things to remember:The first-line empiric treatment for acute cystitis (a lower urinary tract infection) is nitrofurantoin, fosfomycin, or sulfamethoxazole/trimethoprim. An antibiotic that has not been used within the past 3 months should be selected.

JV is a 46-year-old male who comes to the hospital with cough, blood-tinged sputum, night sweats, and unintentional weight loss of 15 lbs over the past 2 months. A chest x-ray shows a cavitation in the right upper lobe, and Mycobacterium tuberculosis is identified on an induced sputum culture. Which combination drug regimen is the preferred initial therapy for this patient?

C. Isoniazid, ethambutol, pyrazinamide, and rifampin Active tuberculosis (TB) commonly presents with symptoms of cough, hemoptysis (eg, blood-tinged sputum), night sweats, fever, and unintentional weight loss. Objective signs of TB include cavitation on chest x-ray and isolation of Mycobacterium tuberculosis from a respiratory sample. A tuberculin skin test or interferon-gamma release assay blood test are usually positive but must be correlated with symptoms and chest x-ray findings to diagnose active disease. The first-line treatment of active TB is split into two phases with a total duration of at least 6 months. The initial intensive phase consists of a four-drug regimen of rifampin, isoniazid (INH), pyrazinamide, and ethambutol (RIPE) for 2 months. At this time, if initial cultures do not show evidence of antibiotic resistance and repeat sputum cultures are negative, the continuation phase regimen is narrowed to two drugs, rifampin and INH, for 4 months. (Choice A) Bedaquiline, moxifloxacin, and streptomycin are reserved for the treatment of multidrug-resistant TB and are not preferred over RIPE. (Choices B and E) The recommended initial therapy for active TB is a four-drug regimen (eg, RIPE). Pyridoxine (vitamin B6) is used to prevent INH-induced peripheral neuropathy and does not have activity against M. tuberculosis. (Choice D) Although rifampin, isoniazid, and ethambutol are active against M. tuberculosis, pyrimethamine is not. It is commonly used for the treatment of toxoplasmosis and Pneumocystis pneumonia in patients with HIV. Things to remember:Treatment of active tuberculosis consists of an initial intensive phase with a four-drug regimen of rifampin, isoniazid, pyrazinamide, and ethambutol for 2 months, followed by a continuation phase with rifampin and isoniazid for 4 months.

JJ is a 48-year-old male who comes to the urgent care clinic with cough, purulent sputum, and chills. He has not traveled recently and has no known sick contacts. Past Medical History: alopecia, hypertension, type 2 diabetes mellitus, glaucoma, insomnia Allergies: sulfa (hives, anaphylaxis) Medications:Amlodipine 2.5 mg dailyLisinopril 10 mg dailyMetformin 1,000 mg twice dailyLatanoprost 1 drop both eyes at bedtimeMelatonin 3 mg nightly Vital Signs: BP 138/90 mmHg, HR 102 bpm, RR 18 bpm, T 102.6°F (39.2°C) Diagnostic Tests:Chest x-ray: left lower lobe infiltrate consistent with pneumonia Which antibiotic is recommended for the outpatient management of JJ's condition?

C. Levofloxacin The selection of antibiotic treatment for community-acquired pneumonia (CAP) in the outpatient setting is based on an assessment of patient comorbidities (eg, diabetes mellitus), which increase the risk for antibiotic resistance to common CAP organisms and subsequent treatment failure if initial empiric treatment is inadequate. Patients with comorbidities should receive combination treatment with a beta-lactam plus a macrolide or doxycycline, or respiratory fluoroquinolone (eg, levofloxacin) monotherapy. These treatment options are effective in the setting of potential macrolide- or doxycycline-resistant Streptococcus pneumoniae or beta-lactamase-producing bacteria. (Choice A) Amoxicillin (a beta-lactam) has activity against S. pneumoniae and some gram-negative bacteria (eg, Haemophilus influenzae). It is recommended only as monotherapy in healthy patients with CAP. When comorbid conditions are present, the risk for resistant pathogens warrants broader coverage with the addition of a beta-lactamase inhibitor (eg, amoxicillin/clavulanate) and a macrolide or doxycycline. (Choice B) Ciprofloxacin (a fluoroquinolone) is not indicated for CAP because it does not reliably cover S. pneumoniae (ie, it is not a respiratory fluoroquinolone). (Choice D) Teflaro (ceftaroline, a fifth-generation cephalosporin) is an intravenous antibiotic with broad gram-positive [including methicillin-resistant Staphylococcus aureus (MRSA)] and gram-negative coverage. Ceftaroline does not cover atypical organisms but can be used to treat inpatient CAP when combined with a macrolide or a respiratory fluoroquinolone. (Choice E) Zithromax (azithromycin, a macrolide) covers common CAP organisms, including atypical pathogens. When treating outpatient CAP in patients with comorbidities, it must be combined with a beta-lactam. Things to remember:The outpatient treatment of community-acquired pneumonia in patients with comorbidities should include either a beta-lactam combined with a macrolide or doxycycline, or respiratory fluoroquinolone monotherapy.

AF is a 22-year-old nonpregnant female who comes to the women's health clinic with a 2-day history of increased urinary frequency, flank pain, and chills. A physical exam reveals suprapubic and costovertebral angle tenderness. Past Medical History: none Allergies: no known drug allergies Medications: multivitamin daily Vital Signs: BP 118/78 mmHg, HR 84 bpm, RR 16 bpm, O2 sat 98% (room air), T 101.2°F (38.4°C), Ht 5′ 2″, Wt 122 lbs Diagnostic Tests:Urinalysis:Specific gravity 1.01pH 5.6Protein negativeBlood negativeLeukocyte esterase positiveNitrites positiveBacteria manyWhite blood cells 120 cells/mm3 Additional Information:Urine culture pending; local susceptibility to Escherichia coli: cephalexin 84%, ceftriaxone 95%, gentamicin 93%, levofloxacin 93%, nitrofurantoin 97%, sulfamethoxazole/trimethoprim 73% Which empiric treatment is appropriate for this patient?

C. Levofloxacin 750mg po daily x 5 days This patient has signs and symptoms of pyelonephritis, which include flank pain, costovertebral angle tenderness, fever, and a positive urinalysis. Symptoms of cystitis, such as dysuria, increased urinary frequency, or suprapubic tenderness, may also be present. Escherichia coli is the most common cause of pyelonephritis. When available, local susceptibility patterns should be used to determine appropriate empiric therapy while awaiting culture and susceptibility results. Patients healthy enough to be treated as outpatients can receive oral antibiotics, such as a fluoroquinolone (ciprofloxacin or levofloxacin) for 5-7 days or sulfamethoxazole/trimethoprim (SMX/TMP) for 7-10 days. (Choice A) Beta-lactams (eg, cefdinir) are not first-line treatment for pyelonephritis because of inferior efficacy compared to alternatives. If prescribed (eg, resistance to preferred options), beta-lactams require a longer duration of therapy (10-14 days). (Choices B and D) Although fosfomycin and nitrofurantoin achieve high urine concentrations in the treatment of cystitis, they do not obtain the systemic concentrations needed to treat an upper urinary tract infection such as pyelonephritis. (Choice E) When local E. coli susceptibility to SMX/TMP is less than 80% (ie, more than 20% of isolates are resistant), as in this case, SMX/TMP should not be used for empiric therapy. In addition, a duration of 3 days is appropriate for cystitis, but pyelonephritis requires a longer treatment duration. Things to remember:Pyelonephritis is an upper urinary tract infection that presents with flank pain, fever, and costovertebral angle tenderness with or without symptoms of cystitis (eg, dysuria). Oral levofloxacin or sulfamethoxazole/trimethoprim are preferred empiric treatment options (if appropriate based on local susceptibility patterns).

AT is a 25-year-old male who comes to the emergency department with abdominal pain that began 48 hours ago. The pain is localized to the right lower quadrant, is currently described as severe and sharp, and has resulted in two episodes of vomiting in the past 6 hours. The patient has no chronic medical conditions and has had no recent surgeries or hospitalizations. Allergies: penicillin (anaphylaxis) Vital Signs: BP 110/62 mmHg, HR 92 bpm, RR 18 bpm, T 103.1°F (39.5°C), pain score 9/10 Abdominal Examination: guarding and rebound tenderness noted in the right lower quadrant Laboratory Tests:White blood cells 16,400 cells/mm3C-reactive protein 20 mg/dLSerum creatinine 1.1 mg/dLBlood urea nitrogen 25 mg/dL Diagnostic Tests:CT scan of the abdomen with contrast: fluid-filled abscess in the right lower quadrant Additional Information: Intra-abdominal infection caused by a perforated appendix. Schedule percutaneous drainage of abscess and send fluid sample for Gram stain and culture. Start intravenous fluids and antibiotics. Which antibiotic regimen is most appropriate for this patient?

C. Levofloxacin plus metronidazole Symptoms of vomiting, right lower quadrant abdominal pain, fever, and leukocytosis are consistent with acute appendicitis, a type of intra-abdominal infection with a polymicrobial etiology. When a fluid-filled abscess (eg, perforated appendix) is evident on abdominal CT scan, source control (eg, abscess drainage) and broad-spectrum antibiotics targeting streptococci, Enterobacteriaceae (eg, Escherichia coli, Klebsiella spp., Proteus spp.), and anaerobes (eg, Bacteroides fragilis) are required. Community-acquired intra-abdominal infections can be treated with metronidazole (for anaerobic coverage) plus either a fluoroquinolone (eg, levofloxacin) or a second- or third-generation cephalosporin (eg, ceftriaxone, cefotaxime). Ertapenem monotherapy is an alternate option. A fluoroquinolone-based regimen is preferred in patients with a severe beta-lactam allergy (eg, anaphylaxis). (Choices A, B, and D) Beta-lactams (eg, cefepime, cefotaxime, piperacillin/tazobactam) should be avoided in patients with a severe penicillin allergy due to the risk for cross-reactivity. In addition, cefepime and cefotaxime do not provide the needed coverage of anaerobic pathogens and must be used in combination with metronidazole. Antibiotics with antipseudomonal activity (eg, cefepime, piperacillin/tazobactam) are generally reserved for patients with healthcare-acquired infections or risk for resistance. (Choice E) Because vancomycin has activity against only gram-positive pathogens (eg, streptococci, staphylococci) and metronidazole targets only anaerobic organisms, this regimen lacks the essential activity against gram-negative bacilli (eg, Enterobacteriaceae). The inclusion of vancomycin in the treatment regimen is typically restricted to patients at risk for intra-abdominal infections caused by methicillin-resistant Staphylococcus aureus. Things to remember:Intra-abdominal infections, including acute appendicitis, require broad-spectrum antibiotics with activity against streptococci, gram-negative bacilli (eg, Escherichia coli, Klebsiella spp.), and anaerobes. In patients with a severe beta-lactam allergy, metronidazole plus a fluoroquinolone (eg, levofloxacin) is a preferred option.

PT is a previously healthy 32-year-old female who comes to the emergency department with severe pain in her right lower leg. She reports getting an infection from a cut that occurred while hiking in the woods two days ago. The infection has spread quickly and become extremely painful in the last 24 hours. Allergies: NKDA Home Medications: none Vital Signs: BP 95/60 mmHg, HR 110 bpm, RR 18 bpm, O2 sat 96% (room air), T 102.4°F (39.1°C), Ht 5′ 4″, Wt 58 kg, pain 10/10 (right lower leg) Physical Exam: right lower extremity with significant erythema without sharp margins, edema extending beyond the border of erythema, and some areas with a purple discoloration of the skin; the area is hardened with pain disproportionate to its appearance Laboratory Tests:Hemoglobin 13.1 g/dLPlatelets 240,000 cells/mm3White blood cells 16,000 cells/mm3Sodium 138 mEq/LChloride 104 mEq/LBicarbonate 18 mEq/LBlood urea nitrogen 25 mg/dLCreatinine 1.2 mg/dLCalcium 8.4 mg/dLGlucose 85 mg/dLLactic acid 2.8 mmol/L Microbiology: blood cultures pending Additional Information: suspected necrotizing fasciitis, consult surgery for urgent tissue debridement and obtain cultures in the operating room, start empiric IV antibiotics

C. Meropenem + vancomycin + clindamycin Necrotizing fasciitis (NF) is a life-threatening skin and soft tissue infection that leads to the destruction of muscle fascia and overlying subcutaneous fat. The initial presentation is similar to cellulitis (eg, erythema, edema), but rapidly progresses to red-purple skin discoloration and necrosis. Other distinguishing characteristics include wooden-hard induration, pain that is more severe than expected based on the appearance, and systemic toxicity (eg, fever, hypotension). NF requires urgent surgical debridement and broad-spectrum antibiotics. The infection can be monomicrobial or polymicrobial; possible pathogens include group A Streptococcus, Staphylococcus aureus [including methicillin-resistant S. aureus (MRSA)], Enterobacteriaceae, and anaerobes (gram-positive and gram-negative). Appropriate empiric treatment regimens to target these organisms include meropenem or piperacillin/tazobactam in combination with vancomycin or daptomycin. Clindamycin is often added empirically to inhibit streptococcal toxin production. (Choice A) Cefepime (a fourth-generation cephalosporin) and vancomycin both lack the necessary activity against anaerobic organisms. (Choices B and E) Piperacillin/tazobactam or ceftriaxone combined with metronidazole do not provide the needed activity against MRSA. Ceftriaxone (a third-generation cephalosporin) is also not a preferred beta-lactam for empiric NF treatment because its activity is too narrow. In addition, because piperacillin/tazobactam has intrinsic activity against anaerobes, there is no benefit to adding metronidazole. (Choice D) Penicillin G aqueous is too narrow for initial empiric therapy but may be warranted for definitive therapy, with clindamycin, if cultures identify a monomicrobial infection with Streptococcus pyogenes or Clostridium spp. Things to remember:Necrotizing fasciitis is a rapidly progressing and life-threatening skin and soft tissue infection. Treatment requires urgent surgical debridement and broad-spectrum antibiotics (meropenem or piperacillin/tazobactam in combination with vancomycin or daptomycin). Clindamycin can be added empirically to suppress streptococcal toxin production.

GM is a 46-year-old female who comes to the emergency department with a 2-day history of decreased appetite, 8 to 10 loose, watery stools per day, and abdominal cramping. She was treated for a nonsevere Clostridioides difficile infection 8 weeks ago with oral vancomycin. Laboratory tests today show a white blood cell count of 16,500 cells/mm3, hemoglobin of 16 g/dL, and a serum creatinine of 0.9 mg/dL. A stool PCR test is positive for Clostridioides difficile. Which treatment is appropriate at this time?

C. Oral Dificid Clostridioides difficile infection (CDI) treatment is categorized based on occurrence (initial, first, and subsequent recurrences) as well as severity of presentation (nonsevere, severe, fulminant). Risk factors for recurrent CDI include previous CDI, severe initial presentation, age > 65, and immunocompromised state. A first CDI recurrence (second episode) is preferably treated with either oral Dificid (fidaxomicin) or a vancomycin tapered and pulsed regimen. An oral vancomycin standard regimen is an alternative option only if metronidazole was used for the first CDI episode. (Choice A) Bezlotoxumab (a monoclonal antibody) binds to and neutralizes C. difficile toxin B. It cannot treat CDI when used as monotherapy but can be administered in combination with recommended antibiotic treatment to prevent CDI recurrence in high-risk patients. (Choice B) Due to reduced efficacy, Flagyl (metronidazole) is indicated only for an initial nonsevere CDI episode when access to fidaxomicin or vancomycin is limited. It is not recommended for recurrent CDI because repeated use increases the risk of neurotoxicity (eg, peripheral neuropathy). (Choice D) Neomycin (an oral aminoglycoside) has poor absorption (3%) and concentrates in the gastrointestinal system. Neomycin is commonly used in preoperative bowel preparation regimens but has no activity against C. difficile. (Choice E) Rifaximin can be used following the completion of a standard vancomycin regimen in patients with a third (or subsequent) CDI episode. Rifaximin is not used as monotherapy for CDI due to reported resistance of 30-50%. Things to remember:Oral fidaxomicin or a vancomycin tapered and pulsed regimen are treatment options for a first recurrence of Clostridioides difficile infection.

A 21-year-old female comes to the clinic for a first prenatal visit. She has mild nausea but otherwise has no symptoms. Her last menstrual period was 6 weeks ago. The patient has no known health concerns and takes no medications. Allergies: penicillin (anaphylaxis) Vital Signs: BP 110/60 mmHg, HR 88 bpm, RR 12 bpm, T 98.6°F (37°C), Ht 64 inches, Wt 52 kg Laboratory Tests:Hemoglobin 12.5 g/dLHIV-1 antibody negativeHIV-2 antibody negativeRapid plasma reagin positiveTreponemal assay positivehCG (urine) positive Which treatment plan is most appropriate for this patient?

C. Penicillin desensitization, then give penicillin G benzathine 2.4 million units IM weekly for 3 weeks Routine screening for HIV and sexually transmitted infections is recommended at the first prenatal visit during pregnancy. A positive nontreponemal test (rapid plasma reagin) and a treponemal assay convey a diagnosis of syphilis, for which the only effective treatment during pregnancy is penicillin G, using a regimen appropriate for the stage of disease. Alternative antibiotic regimens lack sufficient efficacy data, cause fetal abnormalities, or do not adequately cross the placenta and treat fetal infection. For these reasons, any pregnant patient with a documented penicillin allergy should undergo desensitization with subsequent administration of penicillin. For syphilis of unknown duration (eg, late latent syphilis), the preferred regimen is penicillin G benzathine 2.4 million units intramuscularly weekly for 3 weeks. (Choice A) Ceftriaxone has not been proven effective to treat syphilis during pregnancy or prevent congenital syphilis. (Choice B) Doxycycline is generally contraindicated during pregnancy due to its ability to permanently discolor teeth and impair bone growth and development. A 2-week course of doxycycline is an alternative treatment for nonpregnant adults with syphilis and a severe penicillin allergy, if adherence is not a concern. (Choice D) Intravenous penicillin G aqueous is the recommended regimen for neurosyphilis (eg, cerebrospinal fluid abnormalities, headache, confusion, stiff neck). In patients with a penicillin allergy, desensitization is required before administration. (Choice E) Treatment of syphilis is critical during pregnancy to prevent the progression of maternal disease and adverse fetal outcomes, including miscarriage, congenital syphilis, and neonatal death. Things to remember:Pregnant patients with syphilis require treatment with an appropriate regimen of penicillin G. When a penicillin allergy is documented, the only course of action is to desensitize the patient and then administer penicillin.

A 46-year-old male comes to the clinic wanting to start HIV pre-exposure prophylaxis (PrEP). He reports no sexual partners for at least a year but previously had multiple sexual partners with inconsistent condom use. Baseline laboratory results are negative for HIV, gonorrhea, and chlamydia; a VDRL and treponemal assay are positive. A physical exam does not identify any lesions of the mucosa, genitals, rectum, or perianal area. He has no headache, changes to vision, or altered mental status. Which regimen is preferred for this patient?

D. Bicillin L-A 2.4 million units IM weekly x 3 weeks Syphilis infections are characterized by periods of latency and periods of active disease, which also dictate the recommended treatment. This patient has a latent infection, based on positive syphilis serology [Venereal Disease Research Laboratory (VDRL) and treponemal assay] and no clear evidence of primary (eg, chancre), secondary (eg, diffuse rash, lymphadenopathy), or tertiary (eg, cardiovascular or neurologic symptoms) syphilis. As the infection was likely acquired more than 1 year ago (late latent), he should be treated with Bicillin L-A (penicillin G benzathine) 2.4 million units intramuscularly once weekly for 3 weeks. (Choices A and B) Bicillin C-R (a combination of long-acting penicillin G benzathine and short-acting penicillin G procaine) is often dispensed in error in place of Bicillin L-A, but it is not recommended for the treatment of syphilis as it does not provide adequate and sustained levels of penicillin. Bicillin C-R is often used for pharyngitis due to Streptococcus species. (Choice C) The correct dose and route of Bicillin L-A (penicillin G benzathine) is 2.4 million units intramuscularly. It should never be administered intravenously as it is a lipid emulsion and has a boxed warning for cardiopulmonary arrest and death, which has occurred from inadvertent IV administration. (Choice E) Penicillin G aqueous has a short half-life (less than 1 hour) and is indicated for the treatment of neurosyphilis at a dose of 3-4 million units IV every 4 hours for 10-14 days. It is not used to treat any other stage of syphilis. Things to remember:Asymptomatic patients with late latent syphilis (acquired more than 1 year ago) should be treated with penicillin G benzathine 2.4 million units intramuscularly once weekly for 3 weeks.

A 21-year-old male college student is brought to the emergency department by his roommate, who found him in their dorm room lethargic and confused. The patient skipped classes today due to a severe headache and has had one episode of emesis since arriving at the hospital. Social History: does not smoke tobacco, drinks on weekends (beer) Allergies: NKDA Vital Signs: BP 102/64 mmHg, HR 104 bpm, RR 20 bpm, T 102.2°F (39°C), Ht 70″, Wt 181 lbs Laboratory Tests:White blood cells 15,000 cells/mm3Platelets 200,000 cells/mm3Blood urea nitrogen 18 mg/dLSerum creatinine 0.8 mg/dL Diagnostic Tests: lumbar puncture pending Which empiric antibiotic regimen is most appropriate for this patient?

D. Ceftriaxone + Vancomycin Suspected bacterial meningitis (eg, acute onset of severe headache, fever, neck stiffness, and a change in mental status) warrants urgent medical attention, including empiric treatment with IV antibiotics. Antibiotic selection is guided by patient age, immunocompetence, and the common causative pathogens (eg, Streptococcus pneumoniae and Neisseria meningitidis in immunocompetent patients between age 2 and 50). The recommended treatment for community-acquired bacterial meningitis is vancomycin in combination with a third-generation cephalosporin (eg, ceftriaxone, cefotaxime), all of which have adequate cerebrospinal fluid penetration and can improve neurologic outcomes. (Choices A and B) Ampicillin should be included in the empiric regimen when there is a risk of meningitis due to Listeria, which includes neonates (within 1 month of birth), patients greater than age 50, and immunocompromised patients. In neonates, gentamicin provides added activity against Escherichia coli and synergistic effects against Listeria when combined with ampicillin. (Choice C) Sulfamethoxazole/trimethoprim is an alternative to ampicillin when activity against Listeria is needed and the patient has a severe allergy to penicillin (eg, anaphylaxis). However, sulfamethoxazole/trimethoprim does not have reliable activity against S. pneumoniae; vancomycin should be added to the regimen to cover cephalosporin-resistant strains. (Choice E) Moxifloxacin has limited clinical data for bacterial meningitis treatment but is an alternative to a third-generation cephalosporin (eg, ceftriaxone) in patients with a severe beta-lactam allergy. Things to remember:Empiric antibiotic treatment of community-acquired bacterial meningitis in immunocompetent patients age 2 to 50 should consist of vancomycin and a third-generation cephalosporin (eg, ceftriaxone, cefotaxime).

GH is a 64-year-old male who is scheduled for an elective cardiac pacemaker implantation. He has a documented penicillin allergy noted to cause anaphylaxis. Which antibiotic should the operating room pharmacist dispense for administration prior to the procedure?

D. Clindamycin Postoperative surgical site infections (SSIs) develop in 2%-5% of hospitalized patients. Infection risk can be reduced when intravenous antibiotics are correctly timed prior to the start of the procedure and provide the appropriate spectrum of activity for the surgery type (eg, covers the associated local bacterial flora most likely to cause infection). When skin flora (eg, Staphylococcus aureus, coagulase-negative staphylococci, streptococci) are the most likely bacteria to cause an SSI (eg, cardiac procedures), cefazolin is the preferred antibiotic. Patients with a type 1 hypersensitivity reaction (eg, anaphylaxis) to penicillins or cephalosporins should receive intravenous clindamycin (infused 1 hour before the first incision) or vancomycin (infused 2 hours before the first incision). (Choices A and B) Although ampicillin/sulbactam and cefazolin have activity against skin flora, they should not be administered to patients with a history of anaphylaxis to penicillin due to the risk for cross-reactivity. In addition, ampicillin/sulbactam is too broad (covers gram-negative and anaerobic organisms), making it a more appropriate prophylactic antibiotic for gastrointestinal surgeries. (Choice C) Ciprofloxacin is more commonly used prior to genitourinary surgeries (eg, cystoscopy) due to its broad gram-negative activity. With poor activity against gram-positive pathogens (eg, staphylococci), it is not suitable for cardiac surgery prophylaxis. (Choice E) Gentamicin (an aminoglycoside) is used in surgical prophylaxis regimens, in combination with clindamycin or metronidazole, to provide gram-negative activity when first-line antibiotics (eg, cefoxitin, cefazolin) cannot be used (eg, severe penicillin allergy). Things to remember:In patients with a type 1 hypersensitivity reaction (eg, anaphylaxis) to penicillins or cephalosporins, surgical prophylaxis with cefazolin should be avoided. Intravenous clindamycin or vancomycin are the preferred alternatives when targeting skin flora (eg, Staphylococcus aureus, coagulase-negative staphylococci, streptococci).

BC is a 54-year-old male who comes to the clinic for a routine physical. During the visit, the patient mentions that he is scheduled for a root canal with his dentist tomorrow. Past Medical History: gastroesophageal reflux disease, seasonal allergies, multiple dental caries, tooth extraction, endocarditis (1 year ago) Allergies: ampicillin (anaphylaxis) Home Medications:Pantoprazole 40 mg PO dailyIbuprofen 200 mg PO Q4H PRN tooth painCetirizine 10 mg PO dailyFluticasone nasal spray 2 sprays in each nostril daily Vital Signs: BP 122/80 mmHg, HR 76 bpm, RR 16 bpm, O2 sat 98% (room air), T 98.6°F (37°C) Which oral antibiotic regimen is recommended for this patient prior to his planned dental procedure?

D. Doxycycline 100mg This patient with a history of infective endocarditis (IE) requires antibiotic prophylaxis before the scheduled root canal to prevent a future episode of IE. For patients with high-risk conditions (eg, prosthetic heart valve, history of endocarditis), antibiotics are recommended before dental procedures that involve manipulation of gingival tissue or the periapical region of the tooth. The antibiotic kills oral bacteria (eg, viridans group streptococci) that may translocate into the bloodstream during the procedure. The first-line IE prophylaxis treatment is amoxicillin 2 grams PO. Patients with a beta-lactam allergy should receive a macrolide (either azithromycin or clarithromycin 500 mg PO) or doxycycline 100 mg PO. The antibiotic should be administered as a single dose 30-60 minutes before the procedure, which allows time for the medication to be absorbed and distributed into the bloodstream. (Choices A and B) Because this patient has a history of anaphylaxis to ampicillin, beta-lactams, including penicillins (eg, amoxicillin) and cephalosporins (eg, cephalexin), should be avoided due to the risk of cross-reactivity. (Choice C) Clindamycin is not recommended for IE prophylaxis due to a high risk of causing Clostridioides difficile infection. (Choice E) Oral vancomycin is not absorbed systemically (< 5% bioavailability) and therefore cannot kill bacteria that may translocate into the bloodstream during the procedure. Things to remember:Patients with high-risk conditions (eg, prosthetic heart valve, history of endocarditis) should receive antibiotic prophylaxis before dental procedures. For patients with a beta-lactam allergy, the recommended oral options include a macrolide (eg, azithromycin or clarithromycin 500 mg) or doxycycline 100 mg, given as a single dose 30-60 minutes before the procedure.

A patient comes to the clinic with fever, chills, muscle aches and a severe headache. She was recently on a week-long camping trip in South Carolina with her extended family. The patient appears to have a tick bite and is suspected to have Rocky Mountain spotted fever. Rickettsia ricketsii serology is ordered. Which treatment is preferred for this patient?

D. Doxycycline 100mg BID x 7 days Treatment of Rocky Mountain spotted fever involves careful removal of the tick from the skin and antibiotics to eliminate the infection. Doxycycline is the drug of choice and is used for both confirmed and suspected cases in adults and pediatric patients.

BV is a 42-year-old male who comes to his primary care provider with a 2-day history of new-onset shortness of breath, fever, chills, fatigue, and persistent cough. He has no other health issues and no known drug allergies. A chest x-ray confirms pneumonia. The local antibiogram shows Streptococcus pneumoniae is 60% susceptible to macrolides. What is the most appropriate antibiotic therapy for this patient?

D. Doxycycline for 5 days Outpatient community-acquired pneumonia (CAP) treatment should target Streptococcus pneumoniae, Haemophilus influenzae, Moraxella catarrhalis, and atypical pathogens (eg, Mycoplasma pneumoniae). Healthy patients without comorbidities have a lower risk of resistant pathogens, and monotherapy with high-dose amoxicillin or doxycycline is recommended. Although amoxicillin does not have activity against atypical pathogens, it does cover the other likely CAP organisms. A macrolide is an option if local pneumococcal resistance is less than 25%. The recommended duration of treatment for CAP is 5 to 7 days. A duration of treatment longer than 7 days offers no additional benefit and has been associated with increased adverse effects (Choices A, C, and E). (Choices B and C) Azithromycin monotherapy is not a preferred treatment for outpatient CAP in healthy patients without comorbidities if local pneumococcal resistance exceeds 25%. Things to remember:The recommended treatment options for outpatient community-acquired pneumonia in healthy patients with no comorbidities include monotherapy with amoxicillin, doxycycline, or a macrolide (if local pneumococcal resistance is less than 25%). The typical duration of treatment is 5 to 7 days. Longer durations of treatment offer no benefit and increase the risk of adverse effects.

AF is a 22-year-old non-pregnant female with glomerulonephritis and chronic kidney disease. She presents to her primary care provider with nausea, dysuria and flank pain. Her temperature is 100.6°F. A urinalysis is positive for leukocyte esterase, white blood cells and nitrites. A urine culture is pending. Which treatment is recommended for AF?

D. Levaquin AF has acute pyelonephritis. First-line treatment for outpatients is ciprofloxacin or levofloxacin for 5-7 days (unless there is a clear indication that local quinolone resistance rates are >10%). Alternatives include sulfamethoxazole/trimethoprim or a beta-lactam (e.g., amoxicillin/clavulanate, cefdinir), but a longer duration of treatment is required (14 days).

SZ is a 67-year-old female admitted to the hospital for worsening shortness of breath that the patient describes as much worse than her "typical heart failure episodes." Other symptoms include a productive cough with yellow sputum and fever. Past Medical History: chronic obstructive pulmonary disease, heart failure (last EF 30%), hypertension, gastroesophageal reflux disease, peripheral arterial disease Allergies: penicillin (rash and lip swelling) Medications:Entresto 49 mg/51 mg BIDLasix 40 mg BIDCoreg CR 40 mg dailyCombivent Respimat 1 inhalation QIDAldactone 25 mg dailyAspirin 81 mg dailyOmeprazole 20 mg daily Vital Signs: BP 138/91 mmHg, HR 72 bpm, RR 18 bpm, O2 sat 90% on room air, T 101.8°F (38.8°C) Laboratory Tests:White blood cells 13,200 cells/mm3Hemoglobin 12.4 g/dLPlatelets 210,000 cells/mm3 Diagnostic Tests:Chest x-ray: left lower lobe infiltrate Which of the following is the best option for empiric treatment of SZ's acute medical issue?

D. Levofloxacin The clinical presentation of productive cough, fever, shortness of breath, leukocytosis, and an infiltrate on chest x-ray is consistent with community-acquired pneumonia (CAP). The recommended antibiotic regimens target common CAP pathogens (eg, Streptococcus pneumoniae, Haemophilus influenzae, atypicals) and are further based on disease severity, which determines whether the patient is treated in a general medical unit or an intensive care unit (ICU). In patients with a documented severe beta-lactam allergy (eg, angioedema), antibiotics with a risk of cross-reactivity (eg, penicillins) should be avoided. A respiratory fluoroquinolone (eg, levofloxacin) is the best option for the inpatient treatment of nonsevere CAP. (Choices A and E) Although ampicillin/sulbactam plus azithromycin is a treatment option for inpatient CAP, penicillins (ampicillin/sulbactam, piperacillin/tazobactam) should be avoided in patients with a severe penicillin allergy due to the risk of cross-reactivity. In addition, piperacillin/tazobactam has broad-spectrum activity and should be reserved for treating patients with risk factors for Pseudomonas aeruginosa (eg, prior respiratory isolation, hospitalization with administration of IV antibiotics in the past 90 days). (Choices B and C) Azithromycin and doxycycline cover the common CAP organisms, but monotherapy with either antibiotic is recommended only for the outpatient management of CAP in healthy patients without comorbidities (eg, heart failure, diabetes) and is insufficient when treating CAP in the inpatient setting. Things to remember:The preferred inpatient treatment of nonsevere (non-intensive care unit) community-acquired pneumonia in patients with a severe penicillin allergy is a respiratory fluoroquinolone. Penicillins should be avoided if a severe allergy (eg, anaphylaxis, angioedema) is documented.

A 34-year-old female comes to the urgent care clinic with symptoms of burning while urinating as well as increased urinary frequency and urgency. She has no flank pain. Past Medical History: seasonal allergies Social History: drinks 1 or 2 glasses of wine per week Allergies: sulfa (hives) Home Medications: Junel Fe 1/20 1 tab PO once daily, levocetirizine 5 mg PO daily, fluticasone intranasal 2 sprays in each nostril daily Vital Signs: BP 118/78 mmHg, HR 70 bpm, RR 16 bpm, O2 sat 96% (room air), T 98.5°F (36.9°C) Laboratory Tests:White blood cells 9,500 cells/mm3Serum creatinine 0.8 mg/dLUrine hCG negative Diagnostic Tests:Urinalysis:Specific gravity 1.01pH 5.8Protein negativeBlood negativeLeukocyte esterase positiveNitrites positiveBacteria moderateWhite blood cells 100 cells/mm3 Which of the following is an appropriate empiric antibiotic regimen for this patient?

D. Macrobid 100mg PO BID x 5 days Acute cystitis is a lower urinary tract infection (UTI) affecting the bladder that presents with dysuria, suprapubic tenderness, increased urinary frequency, and urgency; signs of pyelonephritis (eg, flank pain) are absent. Cystitis is diagnosed based on the presence of symptoms and a urinalysis that shows pyuria (urine white blood cells > 10 cells/mm3), bacteria, and positive leukocyte esterase and/or nitrites. Urine cultures are recommended for complicated or recurrent UTIs, or if initial treatment fails. Acute cystitis is most commonly caused by Escherichia coli and less commonly by Proteus spp., Klebsiella spp., and Staphylococcus saprophyticus. Treatment recommendations for symptomatic nonpregnant women include oral antibiotics with good urinary penetration, such as: Nitrofurantoin (Macrobid) 100 mg BID × 5 days Fosfomycin 3 grams × 1 dose Sulfamethoxazole/trimethoprim double-strength (Bactrim DS) 1 tablet BID × 3 days (Choice A) Bactrim DS (sulfamethoxazole/trimethoprim) should be avoided in patients with an allergy to sulfa medications. (Choice B) Ceftriaxone (an intravenous third-generation cephalosporin) is too broad-spectrum and is more appropriate for inpatient treatment of pyelonephritis, an upper urinary tract infection that presents with flank pain, costovertebral angle tenderness, and systemic symptoms (eg, fever, hypotension). (Choice C) Clindamycin covers gram-positive aerobes and anaerobes but lacks the necessary activity against gram-negative pathogens to treat acute cystitis. (Choice E) Moxifloxacin does not concentrate in the urine and cannot be used to treat a urinary tract infection. Things to remember:Acute cystitis is a lower urinary tract infection that occurs more often in women and is most commonly caused by Escherichia coli. Oral antibiotic treatment with nitrofurantoin, fosfomycin, or sulfamethoxazole/trimethoprim is recommended for symptomatic nonpregnant patients.

RR is a 67-year-old female who came to the hospital 2 days ago with fever, altered mental status, increased urinary frequency, and flank pain. On admission, a urine culture was performed, and she was started on empiric IV ceftriaxone. Past Medical History: recurrent urinary tract infections, hypertension, hyperlipidemia, obesity Allergies: NKDA Home Medications:Amlodipine 10 mg PO dailyHydrochlorothiazide 25 mg PO dailyLisinopril 5 mg PO dailySaxenda 1.2 mg SC dailyRosuvastatin 20 mg PO dailyCranberry extract 900 mg PO daily Vital Signs: BP 115/76 mmHg, HR 93 bpm, RR 16 bpm, O2 sat 98% (room air), T 101.3°F (38.5°C), Ht 61 inches, Wt 78 kg Laboratory Tests:White blood cells 15,000 cells/mm3Platelets 215,000 cells/mm3Hemoglobin 13.1 g/dLSerum creatinine 1.5 mg/dLBlood urea nitrogen 22 mg/dL Which antibiotic should be recommended to replace ceftriaxone at this time?

D. Meropenem Pyelonephritis is an upper urinary tract infection (UTI) that presents with symptoms of fever, nausea/vomiting, flank pain, and costovertebral angle tenderness with or without symptoms of cystitis (eg, dysuria, increased urinary frequency). Diagnosis is confirmed with a urinalysis showing pyuria (white blood cells > 10 cells/mm3), bacteria, and positive leukocyte esterase and/or nitrites. A urine sample should be collected before initiating empiric antibiotics and sent for culture to aid in streamlining therapy. Escherichia coli is the most common cause of pyelonephritis, followed by other gram-negative pathogens (eg, Proteus, Klebsiella). Patients ill enough to require hospitalization require empiric intravenous antibiotics, such as ceftriaxone or a fluoroquinolone (ciprofloxacin or levofloxacin). Initial therapy should be adjusted based on culture and susceptibility results; carbapenems (eg, ertapenem, meropenem) are the drugs of choice when an extended-spectrum beta-lactamase (ESBL)-producing organism is identified. ESBLs are bacterial enzymes that inactivate most beta-lactam antibiotics (including penicillins, cephalosporins, and aztreonam), making them ineffective. (Choice A) Amikacin (an aminoglycoside) would not be preferred in this elderly patient with an elevated serum creatinine due to the risk of additional injury to the kidney. (Choices B and C) Both cefepime and levofloxacin are reported as resistant on the culture and susceptibility report and therefore will be ineffective. (Choice E) Nitrofurantoin does not achieve adequate systemic concentrations to treat pyelonephritis. Its use is limited to the treatment of cystitis in patients with a creatinine clearance > 60 mL/min. Things to remember:Pyelonephritis is an upper urinary tract infection that presents with fever, nausea/vomiting, flank pain, and costovertebral angle tenderness. Carbapenems are the preferred treatment when extended-spectrum beta-lactamase (ESBL)-producing bacteria are identified.

FT is a 58-year-old female who was admitted to the hospital 7 days ago after a motor vehicle collision. She has been mechanically ventilated since admission. Today, she develops increased respiratory secretions and worsening oxygenation; her most current vital signs, laboratory tests, and imaging are shown below. Allergies: NKDA Vital Signs: BP 110/75 mmHg, HR 105 bpm, RR 24 bpm, O2 sat 96% (FiO2 60%, up from 40%), T 103.1°F (39.5°C) Laboratory Tests: Sodium 140 mEq/L Potassium 3.8 mEq/L Chloride 108 mEq/L Bicarbonate 22 mEq/L Blood urea nitrogen 14 mg/dL Creatinine 1.2 mg/dL Calcium 9.2 mg/dL Glucose 100 mg/dL Lactate 1.6 mmol/L White blood cells 16,000 cells/mm3Hemoglobin 13 g/dL Platelets 280,000 cells/mm3 Diagnostic Tests: Chest x-ray: dense infiltrates in the right lower lobe Bronchoalveolar lavage culture: pending Which of the following empiric antibiotic regimens is most appropriate?

D. Meropenem + tobramycin + vancomycin Ventilator-associated pneumonia (VAP), a nosocomial pneumonia that develops ≥ 48 hours after endotracheal intubation, often stems from the aspiration of oropharyngeal or gastric secretions. Common causative pathogens include gram-negative bacilli (eg, Pseudomonas aeruginosa) and gram-positive cocci [eg, methicillin-resistant Staphylococcus aureus (MRSA)]. Symptoms include fever, purulent secretions, worsening oxygenation, leukocytosis, and chest x-ray evidence of pneumonia (eg, infiltrates). Empiric treatment of VAP consists of an antipseudomonal beta-lactam that also has activity against methicillin-susceptible S. aureus (eg, cefepime, meropenem). In patients with risk factors for multidrug resistance (eg, hospitalization ≥ 5 days before VAP onset, IV antibiotic use within the past 90 days), a second antipseudomonal antibiotic from a separate class (eg, an aminoglycoside) plus a third agent active against MRSA (vancomycin or linezolid) should be added. (Choices A and C) Both ampicillin/sulbactam and ceftriaxone lack activity against Pseudomonas. These antibiotics are more appropriate for community-acquired pneumonia treatment regimens. (Choice B) Although the combination of cefepime and ciprofloxacin for Pseudomonas is appropriate, daptomycin (despite having MRSA activity) cannot treat pneumonia because it is inactivated by pulmonary surfactant. (Choice E) Ceftaroline does not cover Pseudomonas and is not recommended for pneumonia caused by MRSA due to a lack of data. In addition, antibiotics within the same class (eg, two beta-lactams) should not be used together because they are subject to similar mechanisms of resistance and there is increased risk of class-associated toxicities. Things to remember:Ventilator-associated pneumonia is a nosocomial pneumonia that develops ≥ 48 hours after intubation. Empiric treatment should target Pseudomonas aeruginosa and Staphylococcus aureus. Additional antibiotics should be added (a second antipseudomonal agent and therapy for methicillin-resistant S. aureus) when risk factors for multidrug resistance are present.

KM is a previously healthy 20-year-old male who lives in undergraduate housing at a local state university. He comes to the emergency department due to severe headache, fever, stiff neck, and altered mental status. He is diagnosed with community-acquired meningitis. A lumbar puncture is performed, and he is started on empiric antibiotics and dexamethasone. The cerebrospinal fluid Gram stain shows gram-negative diplococci. What is the likely organism causing meningitis in this patient?

D. Neisseria meningitidis The Gram stain is often the first piece of clinical data that provides information about the microorganism causing an infection. It is completed before routine culture techniques and the results help guide the initial empiric antibiotic treatment while awaiting culture results, which can take up to 48 hours. Gram-negative diplococci are indicative of Neisseria meningitidis, the second most common cause of bacterial meningitis in adults. It is highly transmissible via respiratory droplets; immunization against the most common serotypes is recommended for patients at high risk (eg, college-age adults living in university housing, HIV infection, travel to a high endemic area). (Choice A) Escherichia coli is a gram-negative rod (bacillus) that is part of the normal urogenital flora and thus can be a cause of bacterial meningitis in neonates due to exposure during delivery. (Choice B) Haemophilus influenzae is a gram-negative coccobacillus (an intermediary shape between a coccus and bacillus). Due to widespread immunization of children with H. influenzae type b vaccine, it is a rare cause of meningitis. (Choice C) Listeria monocytogenes is a gram-positive rod that is a common cause of meningitis in neonates (< 1 month from birth), patients older than 50 years, and immunocompromised individuals. (Choice E) Streptococcus pneumoniae appears as gram-positive (not gram-negative) diplococci and is the most common cause of bacterial meningitis in adults. Things to remember:The Gram stain is an important technique in narrowing and categorizing the causative pathogen causing an infection. Gram-negative diplococci are indicative of Neisseria meningitidis, a common cause of bacterial meningitis in adults.

A 13-year-old male visits his primary care provider with symptoms of sudden-onset sore throat, chills, and headache. He does not have a cough or runny nose. He is found to have a temperature of 101°F (38.3°C), inflamed tonsils with patchy exudates, and swollen lymph nodes in the neck. A rapid antigen swab of the throat is positive for Streptococcus pyogenes. He has no known drug allergies. Which of the following is a first-line treatment for this patient?

D. Penicillin V potassium Causes of acute pharyngitis include one of several respiratory viruses (eg, adenovirus, rhinovirus, coronavirus) or bacteria, most commonly Streptococcus pyogenes [also referred to as group A Streptococcus (GAS)]. Symptoms of bacterial pharyngitis include sore throat, fever, patchy tonsillar exudates, and prominent, tender, cervical lymphadenopathy. Cough and other respiratory symptoms (eg, rhinorrhea, congestion) are usually absent. A rapid antigen test or throat culture is performed to confirm the presence of GAS. Streptococcal pharyngitis should be treated with penicillin or amoxicillin to prevent complications (eg, rheumatic fever) and transmission of the disease. These agents are preferred due to their narrow spectrum, favorable safety, and efficacy (S. pyogenes resistance has not been identified). (Choice A) Amoxicillin/clavulanate is too broad-spectrum to treat streptococcal pharyngitis because S. pyogenes is highly susceptible to penicillin and amoxicillin. (Choice B) Due to higher rates of gastrointestinal adverse effects and the prevalence of streptococcal resistance to macrolides, erythromycin is not recommended. Azithromycin, which is better tolerated, is an option for patients with a severe beta-lactam allergy (eg, anaphylaxis); due to similar concerns of resistance, it is not first-line. (Choice C) Fluoroquinolones (eg, levofloxacin) are not recommended for bacterial pharyngitis because they are too broad-spectrum and have a higher risk of adverse reactions (eg, tendonitis). (Choice E) Bacterial pharyngitis should be treated with antibiotics to reduce symptom severity and duration, and to prevent further complications. Because it is highly transmissible to close contacts, antibiotic treatment mitigates spread to others. Things to remember:Acute streptococcal pharyngitis presents with sore throat, fever, tonsillar exudates, and swollen cervical lymph nodes. Antibiotic treatment with penicillin or amoxicillin alleviates symptoms, prevents complications, and reduces further spread.

AP is a 66-year-old male with a history of uncontrolled type 2 diabetes (last hemoglobin A1C: 8.6%), hypertension, and hyperlipidemia who reports a two-day history of 8 to 9 loose, watery stools each day, abdominal cramping, and fever. He recently completed a six-week course of ertapenem for osteomyelitis secondary to a diabetic foot ulcer. A stool test is positive for Clostridioides difficile toxin. Which treatment is recommended for this patient?

D. Vancomycin 125mg po qid for x days Clostridioides difficile is a gram-positive, obligate anaerobic, spore-forming rod that can produce toxins that attack the intestinal lining and cause inflammation of the colon (colitis). Infection arises from the ingestion of C. difficile spores paired with a disruption of the normal bacterial gastrointestinal flora (eg, after exposure to antibiotics or chemotherapy). Common symptoms include at least three loose, watery stools per day, abdominal pain, fever, and leukocytosis. The recommended treatment of an initial episode of C. difficile infection consists of either oral fidaxomicin 200 mg BID or oral vancomycin 125 mg QID for 10 days. Various antibiotic regimens are recommended for recurring episodes. (Choice A) Although fidaxomicin is a preferred medication for the treatment of C. difficile, the recommended duration of treatment is 10 days. Longer durations have not been shown to offer clinical benefit. (Choices B and C) Due to lower rates of cure seen in clinical studies, metronidazole should be considered only for the treatment of nonsevere C. difficile infection if neither oral fidaxomicin nor oral vancomycin is available. (Choice E) The recommended dose of oral vancomycin is 125 mg PO QID. The 500 mg dose is reserved for fulminant infections where the gastrointestinal clearance of vancomycin may be increased (due to a high volume of diarrhea), making it difficult to maintain adequate concentrations in the colon. Things to remember:Antibiotic exposure disrupts the normal gastrointestinal flora, and when paired with ingestion of Clostridioides difficile spores leads to symptoms of infection. Initial episodes should be treated with oral fidaxomicin 200 mg BID or oral vancomycin 125 mg QID for a duration of 10 days.

A 31-year-old pregnant female comes to the obstetrician office due to yellow-green, frothy vaginal discharge that has a foul odor. Other symptoms include pain with urination and during sexual intercourse. Vaginal swabs are positive for Trichomonas vaginalis and negative for Chlamydia trachomatis and Neisseria gonorrhoeae. Which treatment is recommended for this patient?

E. Metronidazole 500mg PO BID for 7 days Trichomoniasis is a sexually transmitted disease caused by the anaerobic protozoan Trichomonas vaginalis. The recommended treatment for females is oral metronidazole 500 mg twice daily for 7 days, while males can be treated with a 2-gram single dose. The 2-gram single dose should not be used in females due to a lower cure rate. Patients should be advised to abstain from sex until they and their sex partners have completed therapy and symptoms have resolved. T. vaginalis infection in pregnant females is associated with adverse pregnancy outcomes (eg, preterm delivery). Regardless of pregnancy stage, symptomatic pregnant patients should be tested and treated. Although metronidazole crosses the placenta, clinical data have demonstrated a low risk to the fetus. (Choice A) Azithromycin (a macrolide) does not have activity against T. vaginalis. Azithromycin 1 gram orally as a single dose is the preferred regimen for pregnant patients with chlamydia. (Choice B) Ceftriaxone (a third-generation cephalosporin) has broad activity against gram-positive (eg, streptococci) and gram-negative (eg, Escherichia coli, Klebsiella) bacteria, but not protozoa (eg, T. vaginalis). Ceftriaxone is the recommended treatment for gonorrhea. (Choice C) Clindamycin has activity against aerobic (eg, streptococci, staphylococci) and anaerobic gram-positive organisms, but not T. vaginalis. Clindamycin is a treatment option for bacterial vaginosis. (Choice D) Metronidazole gel does not reach therapeutic levels in the urethra or perivaginal glands and due to lower trichomoniasis cure rates (≤ 50%) compared to oral metronidazole, it is not recommended. Things to remember:Trichomoniasis is a sexually transmitted disease caused by Trichomonas vaginalis. The recommended treatment is oral metronidazole 500 mg twice daily for 7 days (pregnant and nonpregnant females) or a 2-gram single dose (males).

A 57-year-old male comes to his primary care provider due to a painless ulcer on his penis that he first noticed 5 days ago. The patient has no history of sexually transmitted infections but has been sexually active with multiple partners while using condoms inconsistently in the past 12 months. Physical examination confirms the presence of a 2-cm penile chancre and mild swelling of the lymph nodes in the groin but is otherwise unremarkable. A rapid plasma reagin and treponemal test are positive. The patient has no known drug allergies. What is the most appropriate treatment for this patient?

E. Penicillin G benzathine 2.4 million units IM once Sexual activity with multiple partners and improper condom use increase the risk for sexually transmitted infections. Syphilis is a systemic infection caused by the spirochete Treponema pallidum, and the clinical findings at presentation help determine the stage of disease. Classic findings of primary syphilis are a painless ulcer (known as a chancre), typically in the genital area, and localized lymphadenopathy. Both a nontreponemal test, such as a rapid plasma reagin (RPR) or Venereal Disease Research Laboratory (VDRL), and a treponemal test [eg, fluorescent treponemal antibody absorption (FTA-ABS) or T. pallidum enzyme immunoassay] are recommended to confirm a syphilis diagnosis. Primary syphilis is treated with a single dose of intramuscular penicillin G benzathine (Bicillin L-A) 2.4 million units. (Choice A) Azithromycin is not a reliable treatment for syphilis due to T. pallidum resistance to macrolides. (Choice B) Ceftriaxone is not a preferred treatment for syphilis as there is limited data regarding its efficacy. When used, the recommended dose is 1-2 grams IV or IM daily for 10-14 days. A single dose of ceftriaxone IM is the first-line treatment for gonorrhea. (Choice C) Doxycycline is a recommended alternative for the treatment of syphilis in nonpregnant adults with a penicillin allergy, although the duration of treatment for primary syphilis is 14 days. A 7-day course of doxycycline is appropriate as a first-line treatment for chlamydia. (Choice D) Intravenous penicillin G aqueous is recommended only when there is evidence of neurosyphilis (eg, confusion, headache, abnormal CSF results). Intravenous penicillin has more rapid and reliable penetration of the cerebrospinal fluid compared to intramuscular formulations. Things to remember:Penicillin G benzathine 2.4 million units administered as a single intramuscular dose is the first-line treatment for primary, secondary, or early latent syphilis.

CG is a 72-year-old female admitted to the hospital with a 3-day history of 9 to 10 loose watery stools per day, abdominal pain, and confusion. She received clindamycin prior to an outpatient root canal procedure one week ago. On examination, the patient is noted to have abdominal distention. Past Medical History: endocarditis (2 years ago), hypertension, osteoporosis Allergies: penicillin (anaphylaxis) Home Medications: calcium carbonate with vitamin D, hydrochlorothiazide, lisinopril, risedronate Inpatient Medications:Acetaminophen 650 mg PR every 6 hours PRN fever > 38°CLactated Ringer's 1,000 mL IV, infuse 2,000 mL bolus STAT once0.9% sodium chloride 1,000 mL, infuse at 150 mL/hour Vital Signs: BP 82/50 mmHg, HR 110 bpm, RR 22 bpm, T 101.3°F (38.5°C), Ht 5' 4", Wt 60 kg Laboratory Tests:White blood cells 18,000 cells/mm3Hemoglobin 13 g/dLPlatelets 180,000 cells/mm3Sodium 135 mEq/LPotassium 3.4 mEq/LChloride 98 mEq/LBicarbonate 19 mEq/LBlood urea nitrogen 35 mg/dLSerum creatinine 1.6 mg/dLCalcium 8.8 mg/dLGlucose 120 mg/dLLactate 4.5 mmol/L Diagnostic Tests:CT abdomen: diffuse colonic wall thickening and dilatationRectal swab for Clostridioides difficile toxin positive Assessment and Plan: Stooling is now minimal (nurse unable to obtain stool sample), concern for ileus. Hold home medications, continue intravenous fluids, and start targeted antibiotic therapy. Which treatment is most appropriate for this patient?

E. Rectal vancomycin + intravenous metronidazole Fulminant Clostridioides difficile infection (CDI) is a severe presentation characterized by the presence of hypotension or shock, ileus, and/or toxic megacolon. Compared to the treatment of nonfulminant CDI, higher doses of vancomycin (500 mg PO/NG/PR every 6 hours) are recommended in addition to intravenous metronidazole (500 mg every 8 hours). Rectal (PR) vancomycin (eg, prepared as 500 mg in approximately 100 mL of normal saline for administration as a retention enema) is recommended in CDI cases with ileus, which may prevent medications that are administered via the oral or nasogastric routes from reaching the colon (Choices C and D). (Choices A and B) Intravenous vancomycin does not cross into the lumen of the intestinal tract and cannot treat C. difficile. In cases of ileus, oral metronidazole and fidaxomicin are not likely to benefit and may worsen the ileus. In addition, fidaxomicin monotherapy has not been studied in cases of fulminant disease, and metronidazole has lower cure rates compared to vancomycin. Things to remember:Fulminant Clostridioides difficile infection is characterized by hypotension or shock, ileus, and/or toxic megacolon. The recommended treatment is oral, nasogastric, or rectal (if ileus) vancomycin 500 mg every 6 hours plus intravenous metronidazole.

A 62-year-old male was recently admitted to the hospital with spontaneous bacterial peritonitis (SBP). He has no known drug allergies. The infection was treated with cefotaxime for 7 days and the infection resolved. Today, the medical team is implementing a discharge plan. Which antibiotic should be included on the discharge medication list to prevent future episodes of SBP in this patient?

E. Sulfamethoxazole/ trimethoprim Spontaneous bacterial peritonitis (SBP) is an intraabdominal infection that primarily occurs in patients with cirrhosis complicated by ascites. It arises when bacteria (eg, Escherichia coli, Klebsiella pneumoniae) translocate into the peritoneal space. Because approximately 70% of patients who experience an initial episode of SBP will have a recurrence within one year, prevention of subsequent infections is critical. Antibiotic prophylaxis with either sulfamethoxazole/trimethoprim double strength or ciprofloxacin is recommended in all patients after a first episode of SBP to decrease infection recurrence and improve overall survival. In addition, because peritoneal fluid accumulation (ie, ascites) creates an environment that facilitates bacterial proliferation, diuretic therapy (eg, spironolactone and furosemide) should also be implemented. (Choices A, B, C, and D) Clindamycin, dicloxacillin, linezolid, and metronidazole do not have activity against the gram-negative pathogens (eg, E. coli, K. pneumoniae) that are most commonly isolated from ascitic fluid cultures in patients with SBP. Dicloxacillin (an antistaphylococcal penicillin) and linezolid have activity against only gram-positive bacteria (eg, streptococci, staphylococci). Clindamycin targets both gram-positive aerobic and anaerobic organisms. Metronidazole has only anaerobic activity (eg, Bacteroides fragilis). Things to remember:Patients diagnosed with spontaneous bacterial peritonitis should be prescribed antibiotic prophylaxis with sulfamethoxazole/trimethoprim double strength or ciprofloxacin to decrease the risk of recurrence and improve survival.

GT is a 68-year-old male who seeks medical care for a painful bump on the back of his neck. A physical exam shows a 2.5-cm tender red nodule with an overlying pustule and emerging hair, which is determined to be a furuncle. The patient's vital signs are BP 125/80 mmHg, HR 85 bpm, RR 16 bpm, and T 101°F (38.3°C). In addition to incision and drainage, which empiric antibiotic is appropriate for this condition?

E. Sulfamethoxazole/trimethoprim A furuncle is a type of purulent skin and soft tissue infection (SSTI) that involves a hair follicle and presents as a painful, inflamed nodule often referred to as a boil. A carbuncle results when several furuncles coalesce to form a larger inflamed area. Furuncles and carbuncles can arise in any area that contains hair follicles but tend to localize in regions that experience friction (eg, neck, axillae, thighs). Staphylococcus aureus, both methicillin-susceptible (MSSA) and methicillin-resistant (MRSA), is the most common cause of purulent SSTIs (including furuncles). Incision and drainage is a mainstay of treatment; recommended antibiotics target MRSA and MSSA and include doxycycline or sulfamethoxazole/trimethoprim. (Choices A, C, and D) Amoxicillin, penicillin V potassium, and moxifloxacin do not have sufficient activity against S. aureus. Amoxicillin has activity against streptococci, enterococci, and select gram-negative bacteria (eg, Haemophilus, Escherichia coli). Penicillin V potassium has activity against streptococci and enterococci. Moxifloxacin has broad-spectrum activity against gram-positive (eg, Streptococcus pneumoniae), gram-negative (eg, E. coli), and atypical organisms (eg, Mycoplasma pneumoniae). (Choice B) Cefpodoxime (an oral third-generation cephalosporin) has activity against streptococci, MSSA, and gram-negative bacteria (eg, E. coli) but lacks activity against MRSA. Things to remember:Furuncles and carbuncles are purulent skin and soft tissue infections that involve inflammation of the hair follicles. Treatment with incision and drainage is recommended and antibiotics should target both methicillin-susceptible and methicillin-resistant Staphylococcus aureus (eg, sulfamethoxazole/trimethoprim, doxycycline).

A 24-year-old female comes to the pharmacy health clinic with symptoms of nasal congestion, runny nose, and a feeling of pressure in her face for the past 5 days. The nasal drainage is described as yellow and thin. The patient has no fever, cough, shortness of breath, sore throat, myalgia, or headache. She has been using OTC decongestants but is asking for an antibiotic to accelerate resolution of her symptoms. Which of the following treatments is recommended for this patient at this time?

E. Symptomatic relief only The most common cause of acute sinusitis (or rhinosinusitis) is an underlying viral infection (eg, rhinovirus). Viral sinusitis should be differentiated from acute bacterial sinusitis, which is rare (≤ 2% incidence), to avoid unnecessary exposure to antibiotics. Although the clinical presentation (eg, nasal congestion, purulent nasal discharge, facial pressure) is similar, the duration and severity of symptoms can distinguish a viral versus bacterial etiology. Acute bacterial sinusitis is diagnosed when symptoms persist for ≥ 10 days or are severe (temperature > 102°F, purulent nasal discharge, and facial pain) for ≥ 3 days. This afebrile patient with a symptom duration of only 5 days currently meets the criteria for viral sinusitis and antibiotics are not recommended. Continuing symptomatic care (eg, intranasal saline, decongestants) is most appropriate at this time. (Choices A, B, C, and D) Antibiotics are indicated only if sinusitis symptoms worsen or exceed a duration of 10 days. If criteria for antibiotics are met, the first-line treatment is amoxicillin-clavulanate. Doxycycline is an alternative in patients with a penicillin allergy. Amoxicillin, cefdinir, and macrolides (eg, azithromycin) do not provide adequate activity against the likely pathogens (eg, Streptococcus pneumoniae, Haemophilus influenzae, Moraxella catarrhalis) or are less likely to be effective due to pathogen resistance. Things to remember:Acute sinusitis presents with nasal congestion, purulent nasal discharge, and facial pressure; a viral etiology is suspected for illness lasting < 10 days or if classified as nonsevere (eg, temperature < 102°F) for < 3 days. Viral sinusitis is managed symptomatically (eg, intranasal saline, decongestants); if symptoms worsen or continue to persist for ≥ 10 days, antibiotics (eg, amoxicillin/clavulanate) may be prescribed.

CV is a 42-year-old male who comes to his primary care provider with a cough that has persisted for approximately 2-3 weeks. He describes experiencing a "cold" approximately 3 weeks ago, the symptoms of which resolved after a few days except for the acute cough. The cough is productive and accompanied by rib pain but is not associated with any inspiratory "whoops" or vomiting. The patient has no chronic medical conditions or allergies and takes no routine medications. Vital signs and a physical examination are within normal limits except for some diffuse wheezing heard on chest auscultation. Diagnostic TestsCOVID-19 PCR: negativeInfluenza PCR: negative for influenza A and BChest x-ray: no consolidations or infiltrates Which treatment is recommended for this patient?

E. Symptomatic treatment only Acute bronchitis is characterized by a productive or nonproductive cough that persists for 1-3 weeks. Ninety percent of cases are preceded by an upper respiratory tract virus (eg, rhinovirus, coronavirus, influenza). Bacterial causes are rare (< 10%), with Bordetella pertussis being most likely to cause a persistent cough. Because this patient lacks the classic findings (eg, vigorous coughing with inspiratory "whoop" and/or post-tussive emesis), testing and treatment for pertussis are not warranted. If underlying chronic obstructive pulmonary disease (COPD) or objective findings of pneumonia (eg, consolidations or infiltrates on chest x-ray) are absent, bacterial causes (eg, Streptococcus pneumoniae, Haemophilus influenzae, atypicals) can be ruled out. In this case, antibiotics are not recommended; patients should be counseled on symptomatic management (eg, antitussives, expectorants) and the self-limited course of the disease. (Choices A, B, C, and D) Because acute bronchitis typically has a viral etiology, antibiotics have no role in treatment. Amoxicillin/clavulanate (a beta-lactam/beta-lactamase inhibitor) has activity against common respiratory pathogens (eg, S. pneumoniae, H. influenzae) and is a preferred treatment when symptoms are consistent with an acute bacterial exacerbation of COPD (eg, increased dyspnea, sputum volume, sputum purulence). Macrolides (eg, clarithromycin) are the first-line treatment for acute bronchitis confirmed to be caused by Bordetella pertussis (eg, positive nasopharynx swab culture). In addition, monotherapy with a macrolide, doxycycline, or a respiratory fluoroquinolone (eg, levofloxacin) is appropriate when treating outpatient community-acquired pneumonia. Things to remember:Acute bronchitis is diagnosed clinically when cough persists for 1-3 weeks following an upper respiratory tract virus. Because bacterial causes are uncommon, antibiotics are not recommended. The disease is self-limiting and can be managed symptomatically (eg, cough suppressants, expectorants).

SS is a 3-year-old boy with a rash on the left corner of his mouth and around the left side of his nose. The rash first appeared three days ago as a few localized spots. Now there are pustules in some areas and portions of the rash are covered with a golden-colored crust. The patient is otherwise asymptomatic and has no known drug allergies. What is the most appropriate treatment for his condition?

E. Topical mupirocin Impetigo is a superficial skin infection that commonly involves the face and extremities in young children. It begins with papules and pustules that eventually rupture to form hardened golden-yellow (honey-colored)crusts. The lesions tend to stay localized and may be mildly itchy and/or painful. Frequent hand hygiene is recommended because impetigo spreads quickly through touch. The recommended antibiotics for impetigo target the most common bacterial causes (Staphylococcus aureus and Streptococcus pyogenes), reduce transmission, and improve recovery time. Topical antibiotics (eg, mupirocin) are adequate for limited, localized lesions. If there are numerous and/or extensive lesions that make topical therapy impractical, oral cephalexin or dicloxacillin is recommended. (Choices A and B) Topical (not oral) antibiotics are indicated for localized infections. In addition, ciprofloxacin and azithromycin do not have adequate activity against the most likely causative pathogens (S. aureus and S. pyogenes). (Choice C) Topical acyclovir (an antiviral) is indicated for the treatment of recurrent herpes simplex labialis (cold sores), which presents as painful vesicles on the lips. (Choice D) Topical ketoconazole (an antifungal) is used to treat fungal infections on the skin, including tinea corporis (ringworm), which causes expanding ring-shaped plaques with a raised border. Things to remember:Impetigo is a contagious superficial skin infection that affects the face and/or extremities and is most commonly caused by Staphylococcus aureus and group A Streptococcus (S. pyogenes). Limited and localized infections can be treated with topical antibiotics (eg, mupirocin), whereas more extensive lesions require oral therapy (eg, cephalexin, dicloxacillin).

A 68-year-old male comes to the emergency department with an 8-day history of fever, chills, and malaise. Four weeks ago, he underwent a tooth extraction. Past Medical History: type 2 diabetes (last hemoglobin A1C 7.6%), hypertension, dyslipidemia, peripheral neuropathy, mitral valve prolapse Social History: does not use tobacco, alcohol, or recreational drugs Allergies: NKDA Vital Signs: BP 115/75 mmHg, HR 96 bpm, RR 18 bpm, O2 sat 95% (room air), T 101.3°F (38.5°C) Laboratory Tests:Creatinine 1.2 mg/dLWhite blood cells 15,600 cells/mm3Erythrocyte sedimentation rate 67 mm/hr Diagnostic Tests:Chest x-ray: normalEchocardiogram: moderate mitral regurgitation and a 6-mm mobile mass attached to the mitral valveBlood culture: gram-positive cocci in chains, pending identification Additional Information: suspected infective endocarditis; start empiric ceftriaxone and vancomycin Which organism will most likely be identified on the blood culture?

E. Viridans streptococci Infective endocarditis (IE) is most commonly caused by gram-positive organisms, including viridans group streptococci, Staphylococcus aureus, and Enterococcus spp. Staphylococci (including coagulase-negative staphylococci) are particularly prone to causing prosthetic valve endocarditis due to colonization of indwelling or implanted foreign material. IE treatment targets the specific organism isolated on culture and requires a prolonged course (eg, 4-6 weeks) of intravenous antibiotics. Enterococci and streptococci are both round in shape (cocci) and form pairs (diplococci) or short chains. They are indistinguishable under a microscope and must be differentiated by other means (eg, hemolytic properties on blood agar). Staphylococci are round and group together to form clusters. The coagulase test distinguishes S. aureus (which is coagulase positive) from coagulase-negative staphylococci (eg, S. epidermidis). (Choice A) Haemophilus influenzae is a gram-negative coccobacillus that is a common cause of respiratory infections, including community-acquired pneumonia. (Choice B) Listeria monocytogenes is a gram-positive rod (bacilli). It can cause bacterial meningitis in neonates, patients age > 50, and immunocompromised patients. (Choices C and D) Both S. aureus and S. epidermidis can cause IE, but they appear as gram-positive cocci in clusters (not chains) on a Gram stain. S. aureus is also a common cause of purulent skin and soft tissue infections. S. epidermidis is often considered to be a contaminant (part of normal skin flora) unless repeated cultures are positive. Things to remember:Gram-positive organisms (eg, viridans group streptococci, enterococci, staphylococci) are the most common cause of infective endocarditis. Under a microscope, enterococci and streptococci appear as cocci in pairs or chains and staphylococci appear as cocci in clusters.


Conjuntos de estudio relacionados

HIPAA and Privacy Act Training -JKO

View Set

Ch 2 The Constitution True/False

View Set